INSIGHTSIAS SIMPLYFYING IAS EXAM PREPARATION

INSTA 75 Days REVISION PLAN

UPSC Prelims 2020

INSTA Tests 29 to 32 (GS)

KEY & EXPLANATIONS

www.insightsonindia.com prelims.insightsonindia.com | mains.insightsonindia.com Telegram: insightsIAStips | FB: insightsonindia | TW: vinaygb | YT: INSIGHTS IAS BENGALURU | DELHI | HYDERABAD

Copyright © by Insights IAS All rights are reserved. No part of this document may be reproduced, stored in a retrieval system or transmitted in any form or by any means, electronic, mechanical, photocopying, recording or otherwise, without prior permission of Insights IAS.

Insta 75 Days Revision Plan for UPSC Civil Services Prelims – 2020

This document is the compilation of 100 questions that are part of InsightsIAS famous INSTA REVISION initiative for UPSC civil services Preliminary examination – 2020 (which has become most anticipated annual affair by lakhs of IAS aspirants across the country). These questions are carefully framed so as to give aspirants tough challenge to test their knowledge and at the same time improve skills such as intelligent guessing, elimination, reasoning, deduction etc – which are much needed to sail through tough Civil Services Preliminary Examination conducted by UPSC.

These questions are based on this INSTA Revision Plan which is posted on our website (www.insightsonindia.com). Every year thousands of candidates follow our revision timetable – which is made for SERIOUS aspirants who would like to intensively revise everything that’s important before the exam.

Those who would like to take up more tests for even better preparation, can enroll to Insights IAS Prelims Mock Test Series – 2020 (https://prelims.insightsonindia.com). Every year toppers solve our tests and sail through UPSC civil services exam. Your support through purchase of our tests will help us provide FREE content on our website seamlessly.

Wish you all the best!

Team InsightsIAS

INSTA 75 Days REVISION PLAN for Prelims 2020 - InstaTests

DAY – 29 (InstaTest-29)

1. Consider the following statements regarding Codex Alimentarius Commission (CAC) 1. Codex Alimentarius Commission (CAC) is an intergovernmental body with more than 180 members. 2. It was jointly established by the Food and Agriculture Organization of the United Nations (FAO) and the World Health Organization (WHO) Which of the statements given above is/are correct? (a) 1 only (b) 2 only (c) Both 1 and 2 (d) Neither 1 nor 2

Solution: C

• The Codex Alimentarius Commission (CAC) is an intergovernmental body with over 180 members, within the framework of the Joint Food Standards Programme established by the Food and Agriculture Organization of the United Nations (FAO) and the World Health Organization (WHO), with the purpose of protecting the health of consumers and ensuring fair practices in the food trade. • It was established in 1960’s. • The CODEX ALIMENTARIUS international food standards, guidelines and codes of practice contribute to the safety, quality and fairness of this international food trade. Consumers can trust the safety and quality of the food products they buy and importers can trust that the food they ordered will be in accordance with their specifications. http://www.fao.org/fao-who-codexalimentarius/about-codex/en/

2. Nuakhai, a harvesting festival is celebrated in (a) Nagaland (b) Sikkim (c) Odisha (d) Jharkhand

www.insightsonindia.com 1 Insights IAS INSTA 75 Days REVISION PLAN for Prelims 2020 - InstaTests

Solution: C

• Nuakhai: Harvesting festival of Odisha • The festival of nuakhai is a festival to celebrate newly harvested food by the farmers. • The festival traces its origin to the Vedic period where the sages or Rishis used to talk about Panch yajna. One among them was Pralambana yajna which means the cutting of new crops and offering them to mother goddess as followed in Nuakhai festival. • It is also called Nuakhai Parab or Nuakahi Bhetghat.

3. Consider the following statements regarding Sardar Vallabhbhai Patel 1. Rashtriya Ekta Diwas was observed on 31st October and marks the occasion of the birth anniversary of Sardar Vallabhbhai Patel. 2. Operation Polo was launched to liberate and integrate Hyderabad with . Which of the statements given above is/are correct? (a) 1 only (b) 2 only (c) Both 1 and 2 (d) Neither 1 nor 2

Solution: C

Rashtriya Ekta Diwas

• Rashtriya Ekta Diwas was observed on 31st October across the nation. It marks the occasion of the birth anniversary of Sardar Vallabhbhai Patel. Background:

• The government, in 2014, decided to observe Sardar Patel Jayanti Day as Ekta Diwas. This occasion provides an opportunity to re-affirm the inherent strength and resilience of the nation to withstand the threats to its unity, integrity and security. About Sardar Vallabhai Patel:

• Sardar Patel is credited with uniting all 562 princely states in pre-independent India to build the Republic of India.

www.insightsonindia.com 2 Insights IAS INSTA 75 Days REVISION PLAN for Prelims 2020 - InstaTests

Role in the Indian National Movement:

• 1917– Elected as the Secretary of the Gujarat Sabha, the Gujarat wing of the Indian National Congress. • 1918– Led a massive “No Tax Campaign” that urged the farmers not to pay taxes after the British insisted on tax after the floods in Kaira. His effort to bring together the farmers of his area brought him the title of ‘Sardar’. • Supported the non-cooperation Movement launched by Gandhi and Patel toured the nation with him. • 1928– When the lands of farmers were seized after they refused to pay the extra tax to the government, Patel helped the farmers by striking a deal between the government and farmers’ representatives. • 1930– Imprisoned for participating in the famous Salt Satyagraha movement initiated by Mahatma Gandhi. • 1931- Elected as the President of Indian National Congress in its Karachi session where the party deliberated its future path. • Patel was also compelled to use coercion by launching ‘Operation Polo’ to liberate and integrate Hyderabad after the Nizam of Hyderabad entertained false hopes of either joining Pakistan or remaining independent.

4. Which of the following wildlife sanctuaries and National Parks are located in Assam? 1. Nameri National Park 2. Deepor Beel Bird Sanctuary 3. Gorumara National Park 4. Manas National Park Select the correct answer using the code given below (a) 1 and 2 only (b) 1, 2, 3 and 4 (c) 2 and 4 only (d) 1, 2 and 4 only

Solution: D

• Nameri National Park, Manas National Park, Kaziranga National Park, Orang National Park and Deepor Beel Bird Sanctuary are located in Assam. • Gorumara National Park is located in West Bengal. https://www.thehindu.com/sci-tech/energy-and-environment/assam-floods-why-we-need- to-act-fast-to-save-kaziranga-and-its-wildlife/article28720848.ece www.insightsonindia.com 3 Insights IAS INSTA 75 Days REVISION PLAN for Prelims 2020 - InstaTests

5. Consider the following statements regarding Guru Gobind Singh 1. Guru Gobind Singh was the 10th Sikh guru. 2. He founded the principles of Khalsa or the Five ‘K’s. 3. He is known for the introduction of the turban to cover hair. Which of the statements given above is/are correct? (a) 1 and 2 only (b) 2 and 3 only (c) 1 and 3 only (d) 1, 2 and 3

Solution: D

Guru Gobind Singh

• Guru Gobind Singh was the 10th Sikh guru. • He was born at Patna, Bihar, India, on December 22, 1666. His birthday sometimes falls either in December or January or even both months in the Gregorian calendar. The annual celebration of the Guru’s birthday is based on the Nanakshahi calendar. • He became the Sikh guru at the age of nine, following the demise of father, Guru Tegh Bahadur, the ninth Sikh Guru. • He is known for his significant contributions to the Sikh religion, including the introduction of the turban to cover hair. • He also founded the principles of Khalsa or the Five ‘K’s. They are: Kesh (uncut hair), Kangha (a wooden comb for the hair), Kara (an iron bracelet), Kachera (100% cotton tieable undergarment) (not an elastic one) and Kirpan (an iron dagger large enough to defend oneself). • He is also responsible to establish the highest order in the Sikh community. • Followers of the Sikh faith religiously follow the morals and codes of discipline set up by Guru Gobind Singh. • He fought against the Mughals later in battle of Muktsar in 1705. • He was assassinated in 1708. • He named Guru Granth Sahib, the religious text of the Khalsas and the Sikhs, as the next Guru of the two communities.

6. Consider the following statements regarding Humayun’s Tomb 1. It was commissioned by Akbar. 2. It is a UNESCO World Heritage Site.

www.insightsonindia.com 4 Insights IAS INSTA 75 Days REVISION PLAN for Prelims 2020 - InstaTests

Which of the statements given above is/are correct? (a) 1 only (b) 2 only (c) Both 1 and 2 (d) Neither 1 nor 2

Solution: B

About Humayun’s Tomb, Delhi:

• This tomb, built in 1570, is of particular cultural significance as it was the first garden- tomb on the Indian subcontinent. Commissioned by Humayun’s first wife and chief consort, Empress Bega Begum (also known as Haji Begum). Humayun’s garden-tomb is an example of the charbagh (a four-quadrant garden with the four rivers of Quranic paradise represented), with pools joined by channels. It is also called the ‘dormitory of the Mughals’ as in the cells are buried over 150 Mughal family members. It is a UNESCO World Heritage Site.

7. Consider the following pairs Navigation Developed/Owned System by 1. GALILEO : United Kingdom 2. GLONASS : Russia 3. QUASI-ZENITH : China Which of the pairs given above is/are matched correctly? (a) 2 only (b) 1 only (c) 1 and 2 only (d) 2 and 3 only

Solution: A

• GLONASS is an acronym, which stands for Globalnaya Navigazionnaya Sputnikovaya Sistema, or Global Navigation Satellite System. GLONASS is Russia’s version of GPS (Global Positioning System). www.insightsonindia.com 5 Insights IAS INSTA 75 Days REVISION PLAN for Prelims 2020 - InstaTests

List of Global Navigation Satellite Systems:

• GPS of the United States of America. • Galileo of the European Union. • IRNSS or NAVIC of India. • Quasi-Zenith Satellite System (QZSS) of Japan

8. Under Buddhism, Abhaya Mudra indicates (a) Gesture of reassurance, blessing and protection. (b) Gesture of Charity (c) To invoke earth as witness to the truth (d) Gesture of threat and warning

Solution: A

www.insightsonindia.com 6 Insights IAS INSTA 75 Days REVISION PLAN for Prelims 2020 - InstaTests

9. Consider the following statements regarding Kala Kumbh 1. Its objective is to promote Geographical Indication (GI) crafts and heritage of India. 2. It is organized by Ministry of Culture. Which of the statements given above is/are correct? (a) 1 only (b) 2 only (c) Both 1 and 2 (d) Neither 1 nor 2

Solution: A

Kala Kumbh:

• With an objective to promote Geographical Indication (GI) crafts and heritage of India, the Ministry of Textiles is organising Kala Kumbh – Handicrafts Thematic Exhibition in various parts of the country through the Office of Development Commissioner (Handicrafts). • The exhibitions are sponsored by the Export Promotion Council for Handicrafts (EPCH). EPCH was established under Companies Act in the year 1986-87 and is a non- profit organisation, with an object to promote, support, protect, maintain and increase the export of handicrafts.

10. Consider the following statements 1. Haqqani network is a Syria based guerrilla insurgent group using asymmetric warfare to fight against US-led NATO forces in Syria and Iraq. 2. Boko Haram is a jihadist terrorist organization based in Nigeria. www.insightsonindia.com 7 Insights IAS INSTA 75 Days REVISION PLAN for Prelims 2020 - InstaTests

Which of the statements given above is/are correct? (a) 1 only (b) 2 only (c) Both 1 and 2 (d) Neither 1 nor 2

Solution: B

• The Haqqani network is an Afghan guerrilla insurgent group using asymmetric warfare to fight against US-led NATO forces and the government of Afghanistan.

• The Islamic State in West Africa or Islamic State’s West Africa Province, formerly known as Jamā’at Ahl as-Sunnah lid-Da’wah wa’l-Jihād and commonly known as

www.insightsonindia.com 8 Insights IAS INSTA 75 Days REVISION PLAN for Prelims 2020 - InstaTests

Boko Haram until March 2015, is a jihadist terrorist organization based in northeastern Nigeria, also active in Chad, Niger and northern Cameroon

11. Consider the following statements regarding Jain Councils 1. The First Jain Council was held in the 3rd century B.C. under the leadership of Sthulabhadra at Pataliputra. 2. The Second Council was held at Vallabhi in the 5th century A.D under the leadership of Devridhigani. Which of the statements given above is/are correct? (a) 1 only (b) 2 only (c) Both 1 and 2 (d) Neither 1 nor 2

www.insightsonindia.com 9 Insights IAS INSTA 75 Days REVISION PLAN for Prelims 2020 - InstaTests

Solution: C

Jain Councils

• According to a Jain tradition, an oral sacred literature had been passed down from the days of Mahavira, but Bhadrabahu was the last person to know it perfectly. On Bhadrabahu’s death, Sthulabhadra called a Great Council at Pataliputra which was the First Jain Council held in the beginning of third century B.C. It resulted in the compilation of 12 Angas to replace the former 14 Purvas. The Digambaras rejected this canon and declared that the original one was lost. Thus, there was a great urgency to devise new scriptures. • The Second Council was held at Vallabhi (Gujarat) in the 5th century A.D. under the leadership of Devridhigani. It resulted in final compilation of 12 Angas and 12 Upangas.

12. Consider the following statements regarding Zoroastrianism 1. This religion has its origin in Persia by the prophet Zarathustra around 6-7 BCE. 2. Their sacred text is called Zend Avesta is written in Persian. 3. They are a monotheistic religion who believe in one eternal god. Which of the statements given above is/are correct? (a) 1 and 2 only (b) 2 and 3 only (c) 1 and 3 only (d) 1, 2 and 3

Solution: C

Zoroastrianism

• This religion has its origin in Persia by the prophet Zarathustra around 6-7 BCE. They are a monotheistic religion who believe in one eternal god whose name is Ahura Mazda who is the epitome of just behaviour and goodness. There is a spirit of malice and bad behaviour called Angra Mainyu. These two keep fighting with each other and this is an eternal struggle and one day good will win over evil and that will be the last day. • The first contact of the Zoroastrians with India was in 936 A.D. when they fled from Iran because of the Islamic invasions. They are commonly known as Parsis and are presently amongst the smallest (and rapidly shrinking) communities in India. They live

www.insightsonindia.com 10 Insights IAS INSTA 75 Days REVISION PLAN for Prelims 2020 - InstaTests

mostly in Mumbai, Goa and Ahmedabad. Their fire temples called Atash Bahram are rare and only eight known temples exist in the entire country. • Their sacred text called Zend Avesta is written in Old Avestan and consists of 17 sacred songs (gathas) and the Athuna Vairyo (sacred chant), which was supposedly written by Zarathustra himself. The translations of these texts and compiled glossaries are called Zend. This collection is divided into five parts: 1. Yasna: Worship with ceremony and offerings 2. Videvdat: Laws against the demons 3. Yashts: to worship through praise 4. Khordeh Avesta: Book on Daily prayers 5. Gathas: These are further divided into five parts called the Ahunavaiti, Ushtavaiti, Spenta-Mainyu, Vohu-Khshathra and Vashishta-Ishti. They worship fire but also consider air, water and earth to be sacred elements. They believe that dead matter is a corrupting element to everything and hence, they place the dead bodies in the open to be eaten by the vultures. These open spaces are called ‘Dakhma’ and the vultures that eat them are called ‘Dakhma Nashini’. The only space known in India where they are left are the ‘Towers of Silence’ in Mumbai. Now-a-days, people have even started cremating or burying their dead.

13. Which of the following passes is/are located in Western Himalayas? 1. Khardung La 2. Bara-Lacha-La 3. Zoji – La Which of the statements given above is/are correct? (a) 1 only (b) 1 and 2 only (c) 2 only (d) 1, 2 and 3

Solution: D https://www.lehladakhindia.com/passes/

www.insightsonindia.com 11 Insights IAS INSTA 75 Days REVISION PLAN for Prelims 2020 - InstaTests

14. A Code of Gentoo Laws is a translation of (a) Sangam Literature (b) Srautasutras (c) Dharma sastra (d) Manu smriti

Solution: D

• The Gentoo Code (also known as A Code of Gentoo Laws or Ordinations of the Pundits) is a legal code translated from Sanskrit (in which it was known as vivādārṇavasetu) into Persian by Brahmin scholars; and then from Persian into English by Nathaniel Brassey Halhed, a British grammarian working for the East India Company. Vivādārṇavasetu is a digest of Hindu law in 21 sections (taraṅga) compiled for Warren Hastings by the pandits. The translation was funded and encouraged by Warren Hastings as a method of increasing colonial hold over the Indies. It was translated into English with a view to know about the culture and local laws of various parts of Indian subcontinent. It was printed privately by the East India Company in London in 1776 under the title A Code of Gentoo Laws, or, Ordinations of the Pundits. www.insightsonindia.com 12 Insights IAS INSTA 75 Days REVISION PLAN for Prelims 2020 - InstaTests

It is basically about the Hindu law of inheritance (Manusmriti). The Pandits and the Maulvis were associated with judges to understand the civil law of Hindus and Muslims.

15. The work ‘Mricchakatika’ deals with (a) Love affair of young brahmin Charudatta with a wealthy courtesan. (b) Love story of Vikram and Urvasi (c) Story of Duryodhana during and after his fight with Bhima (d) Love story of princess Ratnavali

Solution: A

Kalidasa

• Malavikagnimitra (The love story of Malavika a maiden of Queen and Agnimitra the son of Pushyamitra Shunga) • Vikramorvasiya (Love story of Vikram and Urvasi) • Abhigyana Shakuntala (the recognition of Shakuntala) Sudraka

• Mricchakatika (The Little Clay Cart) Love affair of young brahmin Charudatta with a wealthy courtesan. Vishakhdutta

• Mudra Rakshasa (is a political drama and narrates ascent of king Chandragupta Maurya to power in India) • Devi Chandraguptam Bhavabhuti

• Uttara Ramacharitam (the later life of Rama). It was written in 700 AD. Bhasa

• Swapnavasavadatta (Vasavadatta in dream), Pancharatra, Urubhanga (story of Duryodhana during and after his fight with Bhima. Harshavardhana

• Ratnavali (about the love story of princess Ratnavali, daughter of the king of Ceylon and king Udayana. We find here, the mention of celebration of Holi for the first time).

www.insightsonindia.com 13 Insights IAS INSTA 75 Days REVISION PLAN for Prelims 2020 - InstaTests

16. Consider the following statements regarding Canary Islands 1. It is located in the Mediterranean Sea 2. The Canary Islands are a French archipelago. Which of the statements given above is/are correct? (a) 1 only (b) 2 only (c) Both 1 and 2 (d) Neither 1 nor 2

Solution: D

• The Canary Islands are a Spanish archipelago and the southernmost autonomous community of Spain located in the Atlantic Ocean, west of Morocco at the closest point. • The Canary Islands have been considered a bridge between four continents: Africa, North America, South America and Europe.

17. Consider the following statements regarding Zonal Cultural Centres 1. The Ministry of Culture has set up seven ‘Zonal Cultural Centres’ (ZCCs) across the country. 2. The main objective of these Centres is the development, preservation, promotion and dissemination of the traditional arts of the country. Which of the statements given above is/are correct? (a) 1 only

www.insightsonindia.com 14 Insights IAS INSTA 75 Days REVISION PLAN for Prelims 2020 - InstaTests

(b) 2 only (c) Both 1 and 2 (d) Neither 1 nor 2

Solution: C

• The Ministry of Culture has set up seven ‘Zonal Cultural Centres’ (ZCCs) across the country, having headquarters at Patiala, Udaipur, Allahabad, Kolkata, Dimapur, Nagpur and Thanjavur. The main objective of these Centres is the development, preservation, promotion and dissemination of the traditional arts of the country. • To protect, promote & preserve various forms of folk art and culture throughout the country including Chhattisgarh, Jharkhand & Odisha, the Government of India has set up seven Zonal Cultural Centres (ZCCs) with headquarters at Patiala, Nagpur, Udaipur, Prayagraj, Kolkata, Dimapur and Thanjavur. These ZCCs organize various cultural activities & programmes on regular basis all over the country.

18. Consider the following statements regarding Yoga school of Philosophy 1. The yogic techniques of meditation and physical application will lead to the release of the Purusha from the Prakriti and would eventually lead to salvation. 2. This origin of Yoga has been expounded in the Yogasutra of Patanjali. Which of the statements given above is/are correct? (a) 1 only (b) 2 only (c) Both 1 and 2 (d) Neither 1 nor 2

Solution: C

• The Yoga school literally means the union of two major entities. They argue that human being can achieve salvation by combining meditation and physical application of yogic techniques. It is argued that these techniques lead to the release of the Purusha from the Prakriti and would eventually lead to salvation. This origin of Yoga and the school have been expounded in the Yogasutra of Patanjali that is attributed tentatively to the 2nd century BCE.

www.insightsonindia.com 15 Insights IAS INSTA 75 Days REVISION PLAN for Prelims 2020 - InstaTests

• The physical aspect of this school deals primarily with exercises in various postures that are also called asanas. There are several types of breathing exercises that are called pranayams. • The Yoga school favours these techniques as they help humans to control their mind, body and sensory organs. They propound that these exercises can help if one believes in the existence of god as a guide, mentor and teacher. They would help the individual to move away from worldly matter and achieve the concentration required to get salvation.

19. The International IP Index has been released by (a) World Intellectual Property Organization (WIPO) (b) Global Innovation Policy Center (GIPC) (c) World Trade Organization (WTO) (d) Institute of Economics & Peace (IEP)

Solution: B

• The International IP Index 2020 has been released by the U.S. Chamber of Commerce’s Global Innovation Policy Center (GIPC). What is IIP index?

• Released by GIPC, the Index evaluates the IP infrastructure in each economy based on 45 unique indicators, which are critical to the growth of effective IP systems. • The indicators encompass 8 categories of IP protection: patents, copyrights, trademarks, trade secrets, commercialization of IP assets, enforcement, systemic efficiency, and membership and ratification of international treaties. Performance of India:

• Ranked 40 out of 53 global economies. • Last year India was ranked at 36th position out of 50 countries. • India also continues to score well in the Systemic Efficiency indicator, scoring ahead of 28 other economies in these indicators.

20. Consider the following statements regarding Vaisheshika school 1. According to this school, everything in the universe was created by the four main elements: air, water, earth and ether (sky).

www.insightsonindia.com 16 Insights IAS INSTA 75 Days REVISION PLAN for Prelims 2020 - InstaTests

2. They developed the atomic theory and are responsible for the beginning of physics in Indian subcontinent. Which of the statements given above is/are correct? (a) 1 only (b) 2 only (c) Both 1 and 2 (d) Neither 1 nor 2

Solution: B

• The Vaisheshika school believes in the physicality of the Universe and is considered to be the realistic and objective philosophy that governs the universe. • The Kanada who also wrote the basic text governing the Vaisheshika philosophy is often considered the founder of this school. They argue that everything in the universe was created by the five main elements: fire, air, water, earth and ether (sky). These material elements are also called Dravya. They also argue that reality has many categories, for example, action, attribute, genus, inherence, substance and distinct quality. • As this school has a very scientific approach, they also developed the atomic theory, i.e. all material objects are made of atoms. They explain the phenomenon of this universe by arguing that the atoms and molecules combined to make matter, which is the basis for everything that can be physically touched or seen. This school was also responsible for the beginning of physics in Indian subcontinent. • They are considered to be the propounders of the mechanical process of formation of this Universe.

21. Consider the following statements regarding Sophisticated Analytical & Technical Help Institutes (SATHI) 1. It has been launched by Department of Science and Technology 2. It aims to provide professionally managed services with efficiency, accessibility and transparency of highest order under one roof to service the demands of industry, start-ups and academia. 3. The facilities provided by the SATHI shall be utilized by any user/organization on payment of nominal charge. Which of the statements given above is/are correct? (a) 2 only

www.insightsonindia.com 17 Insights IAS INSTA 75 Days REVISION PLAN for Prelims 2020 - InstaTests

(b) 1 and 3 only (c) 1 and 2 only (d) 1, 2 and 3

Solution: D

The Department of Science and Technology launches SATHI Initiative for building shared, professionally managed strong S&T infrastructure. It will address the problems of accessibility, maintenance, redundancy and duplication of expensive equipment in the Institutions. The aim is to provide professionally managed services with efficiency, accessibility and transparency of highest order under one roof to service the demands of industry, start-ups and academia. In the first phase SATHI facilities are being located at IIT- Delhi, IIT-Kharagpur and BHU- Varanasi. The facilities provided by the SATHI may be utilized by any user/organization on payment of nominal charge.

22. Consider the following statements regarding Lokayata Philosophy 1. Brihaspati laid the foundation stone of this school. 2. They argued in favour of Gods and their representatives on the earth. 3. The philosophy is old enough to find mention in the Vedas and Brihadarankya Upanishad. Which of the statements given above is/are correct? (a) 1 and 2 only (b) 2 and 3 only (c) 1 and 3 only (d) 1, 2 and 3

Solution: C

Charvaka School or Lokayata Philosophy

• Brihaspati laid the foundation stone of this school and it was supposed to be one of the earliest schools that developed a philosophical theory. The philosophy is old enough to find mention in the Vedas and Brihadarankya Upanishad.

www.insightsonindia.com 18 Insights IAS INSTA 75 Days REVISION PLAN for Prelims 2020 - InstaTests

• The Charvaka School was the main propounder of the materialistic view to achieve salvation. As it was geared towards the common people, the philosophy was soon dubbed as Lokayata or something derived from the common people. • They argued against Gods and their representatives on the earth – the priestly class. They argued that a Brahman manufactures false rituals so as to acquire gifts (dakshina) from the followers. • Man is the centre of all activities and he should enjoy himself as long as he lives. He should consume all earthly goods and indulge in sensual pleasure. • The Charvakas do not consider ‘ether’ as one of the five essential elements as it cannot be experienced through the perception. Hence, they say the universe consists of only four elements: fire, earth, water and air.

23. The Farmers’ Innovation Fund has been launched by (a) Agricultural and Processed Food Products Export Development Authority (b) Central Soil Salinity Research Institute (c) Commission for Agricultural Costs and Prices (d) The Indian Council of Agricultural Research

Solution: D

• Farmers’ Innovation Fund is the system introduced by the Indian Council of Agricultural Research (ICAR) to scientifically validate, scale-up and propagate the innovations of progressive farmers. The intention was to link farmers and farming with science and to ensure that their farm practices were science-based. Innovations of farmers were already being documented by the Krishi Vigyan Kendras; however, the additional system would encourage farmers to continue their work. As part of this system, an innovation Centre will be established in New Delhi where the innovations will be scientifically validated. As part of efforts to encourage use of technology in farm sector, a linkage had been created between 105 start ups with farmers.

24. Consider the following statements regarding Sangeet Natak Akademi 1. It is an autonomous statutory body funded by Ministry of Culture. 2. It is India’s national academy of music, dance and drama. Which of the statements given above is/are correct? (a) 1 only (b) 2 only www.insightsonindia.com 19 Insights IAS INSTA 75 Days REVISION PLAN for Prelims 2020 - InstaTests

(c) Both 1 and 2 (d) Neither 1 nor 2

Solution: B

Sangeet Natak Akademi, India’s national academy of music, dance and drama, is a pioneer in the creation of modern India. The ephemeral quality of the arts, and the need for their preservation led to the adapting of a democratic system in which the common man had the opportunity to learn, practice and propagate the arts. In 1945, the Asiatic Society of Bengal submitted a proposal for the creation of a National Cultural Trust consisting of three academies-an academy of dance, drama and music; an academy of letters and an academy of art and architecture. It led to the creation of three national academies after Independence. The national Academy named Sangeet Natak Akademi, was the first of these entities to be established by a resolution of the Ministry of Education. In 1961, the Sangeet Natak Akademi was reconstituted by the Government as a society and registered under the Societies Registration Act, 1860 (as amended in 1957).

25. Consider the following statements regarding Bhitarkanika National Park 1. It is located in the state of Odisha 2. It has one of the largest populations of endangered saltwater crocodile in India. 3. It is identified as Ramsar site. Which of the statements given above is/are correct? (a) 2 and 3 only (b) 1 and 3 only (c) 1 and 2 only (d) 1, 2 and 3

Solution: D

The Bhitarkanika National Park is located in Kendrapada district of Odisha in the delta of rivers Brahmani, Baitarani and Dhamara. It is the second largest mangrove ecosystem in India and is a Ramsar site. Gahirmatha Beach and Marine Sanctuary which is known as the world’s largest rookery of Olive Ridley sea turtles, lies to its east.

www.insightsonindia.com 20 Insights IAS INSTA 75 Days REVISION PLAN for Prelims 2020 - InstaTests

The national park is home to saltwater crocodile, White Crocodile, Indian python, King cobra, black ibis etc. It has one of the largest populations of endangered saltwater crocodile in India and is globally unique. Bagagahan, a breeding colony of herons is one of the largest in the country.

DAY – 30 (InstaTest-30)

26. Consider the following statements regarding Coal Bed Methane (CBM) 1. It is an unconventional form of natural gas found in coal deposits or coal seams. 2. It can be used as feedstock for fertilizers 3. It can be used for production of Methanol Which of the statements given above is/are correct? (a) 2 and 3 only (b) 1 only (c) 3 only (d) 1, 2 and 3

Solution: D

Coal Bed Methane (CBM) is an unconventional form of natural gas found in coal deposits or coal seams. CMB is formed during the process of coalification, the transformation of plant material into coal.

• It is considered a valuable energy resource with reserves and production having grown nearly every year since 1989. Varied methods of recovery make CBM a stable source of energy. • CBM can be recovered from underground coal before, during, or after mining operations. It can also be extracted from “unminable” coal seams that are relatively deep, thin or of poor or inconsistent quality. Vertical and horizontal wells are used to develop CBM resources. • Extraction requires drilling wells into the coal seams and removing water contained in the seam to reduce hydrostatic pressure and release absorbed (and free) gas out of the coal.

www.insightsonindia.com 21 Insights IAS INSTA 75 Days REVISION PLAN for Prelims 2020 - InstaTests

CBM can be used for:

• Power generation • As compressed natural gas (CNG) auto fuel • As feedstock for fertilizers • Industrial uses such as in cement production • Methanol production.

27. Consider the following statements regarding Khyal 1. The origin of this style was attributed to Amir Khusrau. 2. Khyal is based on the repertoire of long songs. 3. Usually, the theme for the Khyal bandish is romantic in nature. Which of the statements given above is/are correct? (a) 1 and 2 only (b) 2 and 3 only (c) 1 and 3 only (d) 1, 2 and 3

Solution: C

Khyal

• The word ‘Khyal’ is derived from Persian and means “idea or imagination”. The origin of this style was attributed to Amir Khusrau. This form is popular amongst the artists as this provides greater scope for improvisation. Khyal is based on the repertoire of short songs ranging from two to eight lines. Generally, a Khyal composition is also referred to as a ‘Bandish’. • Sultan Mohammad Sharqi gave the biggest patronage to Khyal in the 15th century. One of the most unique features of Khyal is the use of taan in the composition. Because of this, Alap is given much less room in the Khyal music as compared to Dhrupad. A typical Khyal performance uses two songs: 1. Bada Khyal: sung in the slow tempo 2. Chhota Khyal: sung in the fast tempo In most cases, every singer renders the same bandish differently, while keeping the text and raga as constant. Usually, the theme for these Khyal bandish is romantic in nature. They sing

www.insightsonindia.com 22 Insights IAS INSTA 75 Days REVISION PLAN for Prelims 2020 - InstaTests about love, even if they are related to the divine creatures. It may be praising God or a particular king. Exceptional Khyal compositions are composed in the praise of Lord Krishna.

28. Consider the following statements regarding Hampi 1. It was a part of the Mauryan Empire back in the third century BC. 2. It is a UNESCO world heritage site. Which of the statements given above is/are correct? (a) 1 only (b) 2 only (c) Both 1 and 2 (d) Neither 1 nor 2

Solution: C

Hampi

• Supreme Court backs move of Karnataka government to demolish restaurants near Hampi site. • The court concluded that the constructions were in violation of the Mysore Ancient and Historical Monuments and Archaeological Sites and Remains Act of 1961. About Hampi:

• It is a UNESCO world heritage site. • It was a part of the Mauryan Empire back in the third century BC. • Hampi was the capital city during the four different dynasties altogether in the Vijayanagar city that came into existence in the year 1336 AD. • The Vijayanagara Empire reached unfathomable heights under the guidance of King Krishnadeva Raya of the Tuluva Dynasty. • ‘Kishkindha Kaand’ in Ramayana has special significance concerning Hampi. • It is located near the Tungabhadra river. • By 1500 CE, Hampi-Vijayanagara was the world’s second-largest medieval-era city after Beijing, and probably India’s richest at that time, attracting traders from Persia and Portugal. • It has been described by UNESCO as an “austere, grandiose site” of more than 1,600 surviving remains of the last great Hindu kingdom in South India.

www.insightsonindia.com 23 Insights IAS INSTA 75 Days REVISION PLAN for Prelims 2020 - InstaTests

29. Consider the following statements regarding Earth Hour 1. Earth Hour is a global environmental movement to stop the destruction of nature. 2. Civilians are encouraged to switch off their lights for one hour to help reduce the effect of global warming 3. Earth Hour was started in Earth Summit, 1992. Which of the statements given above is/are correct? (a) 1 only (b) 2 only (c) 1 and 2 only (d) 1 and 3 only

Solution: C

Earth Hour is a global environmental movement to stop the destruction of nature. It is an annual event organized by the World Wide Fund for Nature.

• Civilians are encouraged to switch off their lights for one hour to help reduce the effect of global warming and raise awareness for climate change and wildlife conservation. • Earth Hour started in 2007 when first Earth Hour was held on March 31 in Sydney, Australia at 7:30 pm, local time. Hence Statement 3 is incorrect. • The theme for Earth Hour 2020 – “Climate Action” and sustainable development. https://indianexpress.com/article/lifestyle/life-style/coronavirus-covid19-pandemic-earth- hour-2020-india-sustainable-living-6335605/

30. Markandeshwar temple is located in which of following state? (a) Kerala (b) Andhra Pradesh (c) Tamil Nadu (d) Maharashtra

Solution: D

www.insightsonindia.com 24 Insights IAS INSTA 75 Days REVISION PLAN for Prelims 2020 - InstaTests

Markandeshwar temple

• Known as the “Khajuraho of Vidarbha”, the temple of Markandadeo is situated on the bank of River Wainganga in district Gadchiroli of Maharashtra. • The temples belong to the Nagara group of temples of North India. • On stylistic grounds, their date ranges in between 9- 12th centuries CE. • The temples belong to saiva, vaishnava and sakta faith. Most of the temples have a simple plan, with ardhamandapa, mandapa, antaralaand garbhagriha forming the component of the entire set up.

31. Consider the following pairs of the Indian harvest festivals with the states they are performed 1. Pongal : Karnataka 2. Uttarayan : Gujarat 3. Saaji : Himachal Pradesh Which of the pairs given above is/are correctly matched? (a) 1 only (b) 2 and 3 only (c) 1 and 3 only (d) 1, 2 and 3

Solution: B

Indian harvest festivals

• Makar Sankranti: The festival of Makar Sankranti will be celebrated in Karnataka when the Sun enters the Makar zodiac and the days begin to lengthen compared to nights. • Pongal: In South India and particularly in Tamil Nadu, it’s the festival of Pongal which is being celebrated over 4 days at harvest time. • Magha Bihu: In Assam and many parts of the North East, the festival of Magha Bihu is celebrated. It sees the first harvest of the season being offered to the gods along with prayers for peace and prosperity. • Uttarayan: Gujarat celebrates it in the form of the convivial kite festival of Uttarayan. • Maghi: In Punjab, Makar Sankranti is celebrated as Maghi. Bathing in a river in the early hours on Maghi is important. • Saaji: In Shimla District of Himachal Pradesh, Makara Sankranti is known as Magha Saaji. Saaji is the Pahari word for Sankranti, start of the new month. Hence this day marks the start of the month of Magha.

www.insightsonindia.com 25 Insights IAS INSTA 75 Days REVISION PLAN for Prelims 2020 - InstaTests

• Kicheri: The festival is known as Kicheri in Uttar Pradesh and involves ritual bathing.

32. ‘Operation Save Kurma’, often seen in news, is about conserving (a) Indian Gazelle (b) One horned rhinoceros (c) Turtles (d) Gangetic Dolphins

Solution: C

• Considering the commercial exploitation of the turtles and tortoises as well as the extent of involvement of illegal wildlife traders in the illegal trade of live specimens WCCB envisaged a species specific operation code named “OPERATION SAVE KURMA” to focus specifically on the major states involved in the poaching, transportation and illegal trade of live turtles and tortoises. http://wccb.gov.in/Content/SpecialOperations.aspx

33. Consider the following statements regarding Chindu Yakshaganam 1. It is a form of theatre practised by members of the Chindu Madiga community. 2. It is a form of entertainment in villages across Karnataka. Which of the statements given above is/are correct? (a) 1 only (b) 2 only (c) Both 1 and 2 (d) Neither 1 nor 2

Solution: A

Chindu Yakshaganam

• Chindu Yakshaganam, a.k.a Chindu Bhagavatam is a form of theatre practised by members of the Chindu Madiga community.

www.insightsonindia.com 26 Insights IAS INSTA 75 Days REVISION PLAN for Prelims 2020 - InstaTests

• It is a form of entertainment in villages across Telangana, with the artists skilfully depicting classic tales from the epics and entertaining the masses. • The art form probably started when people enacted acts of hunting, wars and other acts of valour. • The Chindu community has traditionally been a nomadic community.

34. The Hornbill Festival is celebrated in the state of (a) Nagaland (b) Manipur (c) Mizoram (d) Assam

Solution: A

• The tribes of Nagaland celebrate their festivals with appreciation and fervor. More than 60% of the population of Nagaland depends on agriculture and therefore most of their festivals revolve around agriculture. They consider their festivals sacred and so participation in these festivals is essential. To encourage inter-tribal interaction and to promote cultural heritage of Nagaland, the Government of Nagaland organizes the Hornbill Festival every year in the first week of December. Organized by the State Tourism and Art & Culture Departments, Hornbill Festival showcases a mélange of cultural displays under one roof. This festival usually takes place between the 1st and the 7th of December every year in Kohima. • Hornbill Festival is held at Naga Heritage Village, Kisama which is about 12 km from Kohima. All the tribes of Nagaland take part in this festival. The aim of the festival is to revive and protect the rich culture of Nagaland and display its extravaganza and traditions. For visitors it means a closer understanding of the people and culture of Nagaland, and an opportunity to experience the food, songs, dances and customs of Nagaland.

35. Which of the following countries doesn’t open out to the Caspian Sea? (a) Turkey (b) Kazakhstan (c) Iran (d) Russia

www.insightsonindia.com 27 Insights IAS INSTA 75 Days REVISION PLAN for Prelims 2020 - InstaTests

Solution: A

36. Consider the following statements regarding Aryabhatta 1. Aryabhatta wrote Aryabhattiya in which the concepts of mathematics as well as astronomy were mentioned. 2. Aryabhatta in his book stated that the earth is round and rotates on its own axis. 3. Aryabhatta in his book, mentioned Zero for the first time as a number. Which of the statements given above is/are correct? (a) 1 and 2 only (b) 2 and 3 only (c) 1 only (d) 1, 2 and 3

www.insightsonindia.com 28 Insights IAS INSTA 75 Days REVISION PLAN for Prelims 2020 - InstaTests

Solution: A

Aryabhatta in around 499 A.D. wrote Aryabhattiya in which the concepts of mathematics as well as astronomy were explicitly mentioned.

• Aryabhatta in his book stated that the earth is round and rotates on its own axis. He formulated the area of a triangle and discovered algebra. The value of Pi given by Aryabhatta is much more accurate than that given by the Greeks. • The Jyotisa part of Aryabhattiya also deals with astronomical definitions, method of determining the true position of the planets, movement of the Sun and the Moon and the calculation of the eclipses. • Brahmagupta in the 7th century AD in his book Brahmasputa Siddhantika mentioned Zero for the first time as a number. In his book, he also introduced negative numbers and described them as debts and positive numbers as fortunes.

37. Consider the following statements regarding Varahamihira 1. King Vikramaditya awarded him with the title of “Varaha”. 2. The earth cloud theory was propounded by him in his book Brihat Samhita. Which of the statements given above is/are correct? (a) 1 only (b) 2 only (c) Both 1 and 2 (d) Neither 1 nor 2

Solution: C

• In the field of geology, hydrology and ecology, the contribution of Varahamihira cannot be forgotten. He lived in the Gupta period and was among the nine gems in the court of Vikramaditya. His predictions were so accurate that king Vikramaditya awarded him with the title of “Varaha”. He claimed that presence of termites (Deemak) and plants could indicate the presence of water in that particular area. A list of six animals and thirty-six plants was given by him who could indicate the presence of water. • The earth cloud theory was also propounded by him in his book Brihat Samhita. He related earthquake to the influence of plants, behaviour of animals, underground water, undersea activities and the unusual cloud formation. He also made contributions in Astrology or Jyotish shastra.

www.insightsonindia.com 29 Insights IAS INSTA 75 Days REVISION PLAN for Prelims 2020 - InstaTests

38. Consider the following statements regarding Shyama Prasad Mukherji Rurban Mission 1. It is a Centrally Sponsored Scheme 2. It is being implemented by the Union Ministry of Rural Development. 3. The objective is to give a boost to economic activities, skill development and infrastructure amenities in the regions. Which of the statements given above is/are correct? (a) 2 and 3 only (b) 1 and 3 only (c) 1 and 2 only (d) 1, 2 and 3

Solution: D

• 4th anniversary of the Shyama Prasad Mukherji Rurban Mission observed. It was launched from Kurubhat in Rajnandgaon District of Chhattisgarh on 21st February 2016. About Shyama Prasad Mukherji Rurban Mission:

• Launched in 2016, the programme is designed to deliver catalytic interventions to rural areas on the threshold of growth. It is now a Centrally Sponsored Scheme. • It is being implemented by the Union Ministry of Rural Development. • Funding: Shared between the Centre and the State in a ratio of 60:40 for Plain area States and 90:10 for Himalayan and NE States. • The Mission aims at development of 300 Rurban clusters, in five years. • The objective is to give a boost to economic activities, skill development and infrastructure amenities in the regions • Under the mission, the Central government in coordination with the district administration has taken measures to bring about multi-layered phased development of the rural gram panchayats and villages on the lines of urban cities with proper civic amenities keeping the soul of villages intact. What are Rurban areas? Why develop them?

• A ‘Rurban cluster’, would be a cluster of geographically contiguous villages with a population of about 25000 to 50000 in plain and coastal areas and with a population of 5000 to 15000 in desert, hilly or tribal areas. These clusters typically illustrate potential for growth, have economic drivers and derive locational and competitive advantages.

www.insightsonindia.com 30 Insights IAS INSTA 75 Days REVISION PLAN for Prelims 2020 - InstaTests

The larger outcomes envisaged under this Mission are:

• Bridging the rural-urban divide-viz: economic, technological and those related to facilities and services. • Stimulating local economic development with emphasis on reduction of poverty and unemployment in rural areas. • Spreading development in the region. • Attracting investment in rural areas.

39. Consider the following statements regarding Dadasaheb Phalke 1. He is known as the father of Indian cinema. 2. The first motion venture by an Indian was by Dadasaheb Phalke. 3. He produced the film titled Raja Harishchandra and made the first indigenous Indian silent film. Which of the statements given above is/are correct? (a) 1 and 2 only (b) 2 and 3 only (c) 1 and 3 only (d) 1, 2 and 3

www.insightsonindia.com 31 Insights IAS INSTA 75 Days REVISION PLAN for Prelims 2020 - InstaTests

Solution: C

• The first motion venture by an Indian was by Harishchandra Bhatvadekar, who was popularly known as Save Dada. He made two short films in 1899 and exhibited them to the audience by using the Edison Projecting Kinetoscope. • Dadasaheb Phalke who produced the film titled Raja Harishchandra in 1913 made the first indigenous Indian silent film. He is known as the father of Indian cinema and is credited with films like Mohini Bhasmasur and Satyavan Savitri. He is also credited for making the first box office hit titled Lanka Dahan in 1917.

40. Consider the following statements regarding Putul 1. It is a traditional string puppet form of West Bengal. 2. The puppets are about 3 to 4 feet in height and are costumed like the actors of Jatra. 3. These puppets have mostly three joints. Which of the statements given above is/are correct? (a) 2 only (b) 2 and 3 only (c) 1 and 3 only (d) 1, 2 and 3

Solution: B

Putul Nautch, West Bengal

• The traditional rod puppet form of West Bengal is known as Putul Nautch. They are carved from wood and follow the various artistic styles of a particular region. In Nadia district of West Bengal, rod-puppets used to be of human size like the Bunraku puppets of Japan. This form is now almost extinct. The Bengal rod-puppets, which survive are about 3 to 4 feet in height and are costumed like the actors of Jatra, a traditional theatre form prevalent in the State. These puppets have mostly three joints. The heads, supported by the main rod, is joined at the neck and both hands attached to rods are joined at the shoulders. • The technique of manipulation is interesting and highly theatrical. A bamboo-made hub is tied firmly to the waist of the puppeteer on which the rod holding the puppet is placed. The puppeteers each holding one puppet, stand behind a head-high curtain and while manipulating the rods also move and dance imparting corresponding

www.insightsonindia.com 32 Insights IAS INSTA 75 Days REVISION PLAN for Prelims 2020 - InstaTests

movements to the puppets. While the puppeteers themselves sing and deliver the stylized prose dialogues, a group of musicians, usually three to four in numbers, sitting at the side of the stage provide the accompanying music with a drum, harmonium and cymbals. The music and verbal text have close similarity with the Jatra theatre.

41. Consider the following statements regarding HCFC-141b: 1. Recently, India prohibited the export of HCFC-141b. 2. HCFC-141b is used in the production of rigid polyurethane and polyisocyanurate or phenolic insulation foams for residential and commercial buildings Which of the statements given above is/are correct? (a) 1 only (b) 2 only (c) Both 1 and 2 (d) Neither 1 nor 2

Solution: B

India has successfully completed the phasing out of Hydro chloro fluoro carbon (HCFC)- 141b, used by foam manufacturing companies and one of the most potent ozone depleting chemicals.

• HCFC-141b is not produced in India and all the domestic requirements are met through imports. With the notification prohibiting the import of HCFC-141b, India has completely phased out the important ozone depleting chemical. • Simultaneously, the use of HCFC-141b by foam manufacturing industry has also been closed as on January 1 under the Ozone Depleting Substances (Regulation and Control) Amendment Rules, 2014. • The phase out of HCFC-141b from the country has twin environmental benefits, one is assisting the healing of the stratospheric ozone layer and the second is climate

www.insightsonindia.com 33 Insights IAS INSTA 75 Days REVISION PLAN for Prelims 2020 - InstaTests

change mitigation due to transition of foam manufacturing enterprises at large scale to low global warming potential alternative technologies. • Hydrochlorofluorocarbon-141b, or 1,1-dichloro-1-fluoroethane (HCFC141b), has been developed as a replacement for fully halogenated chlorofluorocarbons because its residence time in the atmosphere is shorter, and its ozone depleting potential is lower than that of presently used chlorofluorocarbons. • HCFC-141b is used in the production of rigid polyurethane and polyisocyanurate or phenolic insulation foams for residential and commercial buildings. It may also be used as a solvent in electronic and other precision cleaning applications. • HCFCs are ozone-depleting substances and are to be phased out under the Montreal Protocol by 2020 in the developed world and 2040 elsewhere

42. Consider the following statements regarding Ramman 1. The religious festival is organized once in three years in the twin villages of Saloor- Dungra in the state of Uttarakhand. 2. The festival is organized by villagers, and each caste and occupational group has a distinct role. 3. This event is made up of highly complex rituals and the recitation of a version of the epic of Rama. Which of the statements given above is/are correct? (a) 1 and 2 only (b) 2 and 3 only (c) 1 and 3 only (d) 1, 2 and 3

Solution: B

Ramman, religious festival and ritual theatre of the Garhwal Himalayas, India

• Every year in late April, the twin villages of Saloor-Dungra in the state of Uttarakhand (northern India) are marked by Ramman, a religious festival in honour of the tutelary god, Bhumiyal Devta, a local divinity whose temple houses most of the festivities. This event is made up of highly complex rituals: the recitation of a version of the epic of Rama and various legends, and the performance of songs and masked dances. The festival is organized by villagers, and each caste and occupational group has a distinct role. For example, youth and the elders perform, the Brahmans lead the prayers and perform the rituals, and the Bhandaris – representing locals of the Kshatriya caste – are alone entitled to wear one of the most sacred masks, that of the half-man, half-

www.insightsonindia.com 34 Insights IAS INSTA 75 Days REVISION PLAN for Prelims 2020 - InstaTests

lion Hindu deity, Narasimha. The family that hosts Bhumiyal Devta during the year must adhere to a strict daily routine. Combining theatre, music, historical reconstructions, and traditional oral and written tales, the Ramman is a multiform cultural event that reflects the environmental, spiritual and cultural concept of the community, recounting its founding myths and strengthening its sense of self-worth. In order to ensure that it remains viable, the community’s priorities are to promote its transmission and to obtain its recognition beyond the geographical area in which it is practiced.

43. Consider the following statements regarding Dance 1. Odissi has derived a great deal from the Abhinaya Darpana Prakasha by Jadunatha Sinha. 2. Odissi closely follows the tenets laid down by the . 3. The techniques of movement are built around the two basic postures of the Chowk and the Tribhanga. Which of the statements given above is/are correct? (a) 1 and 2 only (b) 2 and 3 only (c) 1 and 3 only (d) 1, 2 and 3

Solution: D

• Odisha, on the eastern sea coast, is the home of Odissi, one of the many forms of . Sensuous and lyrical, Odissi is a dance of love and passion touching on the divine and the human, the sublime and the mundane. The Natya Shastra mentions many regional varieties, such as the south-eastern style known as the Odhra Magadha which can be identified as the earliest precursor of present day Odissi. • For centuries maharis were the chief repositories of this dance. The maharis, who were originally temple dancers came to be employed in royal courts which resulted in the degeneration of the art form. • Odissi is a highly stylised dance and to some extent is based on the classical Natya Shastra and the Abhinaya Darpana. In fact, it has derived a great deal from the Abhinaya Darpana Prakasha by Jadunatha Sinha, the Abhinaya Chandrika by Rajmani Patra, and the Abhinaya Chandrika by Maheshwara Mahapatra. • Odissi closely follows the tenets laid down by the Natya Shastra. Facial expressions, hand gestures and body movements are used to suggest a certain feeling, an emotion

www.insightsonindia.com 35 Insights IAS INSTA 75 Days REVISION PLAN for Prelims 2020 - InstaTests

or one of the nine rasas. Odissi closely follows the tenets laid down by the Natya Shastra. • The techniques of movement are built around the two basic postures of the Chowk and the Tribhanga. The chowk is a position imitating a square – a very masculine stance with the weight of the body equally balanced. The tribhanga is a very feminine stance where the body is deflected at the neck, torso and the knees.

44. Consider the following pairs of active volcanoes and their respective countries: Active Volcano Country 1. Mount Merapi : Malaysia 2. Taal Volcano : Indonesia 3. Mount Erebus : Sweden 4. Mount Etna : France Which of the pairs given above is/are correctly matched? (a) 1 and 2 only (b) 3 and 4 only (c) 1, 3 and 4 only (d) None

Solution: D

• Mount Merapi is a volcano in Indonesia. It is on the island of Java. It is conical in shape. It is the most active volcano of Indonesia. In Indonesian, the name means mountain of fire. The volcano has been discovered by Mr Morris in 1754. • The Taal Volcano is located about 37 miles (60 kilometers) south of the capital Manila on the island of Luzon and is one of Philippine’s most active volcano. • Mount Erebus is a volcano on Ross Island in Antarctica. It is the most southern active volcano in the world. • Mount Etna is a volcano on the east coast of Sicily, part of southern Italy. It is the largest active volcano in Europe. Mount Etna erupts every few years

45. Consider the following statements regarding Hindu Calendar 1. Hindu Calendar takes into account four angas or limbs, viz., year, month, paksha and tithi. 2. The solar year is divided into two halves and one solar year equals one day and one night of the God. Which of the statements given above is/are correct? www.insightsonindia.com 36 Insights IAS INSTA 75 Days REVISION PLAN for Prelims 2020 - InstaTests

(a) 1 only (b) 2 only (c) Both 1 and 2 (d) Neither 1 nor 2

Solution: B

HINDU CALENDAR

• Panchanga or the Hindu Calendar takes into account panch, i.e. five angas or limbs, viz., year, month, paksha, tithi and ghatika or alternatively, tithi, vaara, nakshatra, yoga and karana. • The twelve places or ecliptic or rashis through which the Sun passes during a year are named after group of stars called Nakshatras. There are 28 nakshatras or constellations in total. Nakshatras being unequal in size do not have the same number of stars, some having even one or two. Each rashi consists of two to three nakshatras. The solar year is divided into two halves under Hindu calendar:

• Uttarayana – First six months from Makara Sankranti to Karka Sankranti ,i.e. from Paush (January) to Ashadh (June) – is the God’s Day. • Dakshinayana – the last six months from July to December is the God’s night. One solar year thus equals one day and one night of the God.

46. Consider the following statements regarding the World Sustainable Development Summit (WSDS): 1. It is the annual flagship event of the International Organization for Sustainable Development (IOSD) 2. Theme of the World Sustainable Development Summit (WSDS), 2020 is “Towards 2030: Making the Decade Count” 3. The WSDS has continued the legacy of the erstwhile Delhi Sustainable Development Summit (DSDS) which was initiated in 2001 with the aim of making ‘sustainable development’ a globally shared goal. Which of the statements given above is/are correct? (a) 1 only (b) 2 and 3 only (c) 1 and 3 only

www.insightsonindia.com 37 Insights IAS INSTA 75 Days REVISION PLAN for Prelims 2020 - InstaTests

(d) 3 only

Solution: B

• The World Sustainable Development Summit (WSDS) is the annual flagship event of The Energy and Resources Institute (TERI). It has in its journey of eighteen years (2001- 2019) become a focal point for global leaders and practitioners to congregate on a single platform to discuss and deliberate over climatic issues of universal importance. The Summit series has emerged as the premier international event on sustainability which focusses on the global future, but with an eye on the actions in the developing world which could bend our common future. • The WSDS has continued the legacy of the erstwhile Delhi Sustainable Development Summit (DSDS) which was initiated in 2001 with the aim of making ‘sustainable development’ a globally shared goal. The Summit series has over the years brought together 47 heads of state and government, 13 Nobel laureates, ministers from 64 countries, 1700 business leaders, 1900+ speakers and over 13,000 delegates from across the world. WSDS 2020 Theme: “Towards 2030: Making the Decade Count”

47. Consider the following pairs of folk dances with the associated community Community 1. Bizu dance : Chakma community 2. Cheraw dance : Hindu Manipuris 3. Basanta Raas : Lusai woman 4. Bamboo Dance : Mizos Which of the pairs given above is/are correctly matched? (a) 1 and 2 only (b) 2 and 3 only (c) 3 and 4 only (d) 1 and 4 only

Solution: D

• ‘Bizu’ dances performed the Chakma community denoting the end of the Bengali calendar year; ‘Hai Hak’ dances of the Halams and the Cheraw dance related with the confinement of Lusai woman www.insightsonindia.com 38 Insights IAS INSTA 75 Days REVISION PLAN for Prelims 2020 - InstaTests

• ‘Basanta Raas’ is the enchanting dance of the Hindu Manipuris, residing in Tripura. Bamboo Dance is the most colourful and distinct dance of the Mizos.

48. Consider the following statements regarding Solar Orbiter Mission 1. It is a cooperative mission between NASA and ISRO 2. The spacecraft will provide images of the Sun’s poles. 3. It will help researchers to know how the star’s energy affects Earth and humans in space Which of the statements given above is/are correct? (a) 1 only (b) 2 and 3 only (c) 1 and 2 only (d) 3 only

Solution: B

• NASA has launched the Solar Orbiter to space on an Atlas 5 rocket from Florida. The mission is a cooperative mission between NASA and the European Space Agency (ESA) which will help researchers to know how the star’s energy affects Earth and humans in space. It a seven-year mission. • It will observe the Sun with high spatial resolution telescopes and capture observations in the environment directly surrounding the spacecraft to know how the Sun can affect the space environment throughout the solar system. It also intends to give a better understanding of how stars can affect the space environment throughout the solar system. The spacecraft also will be the first to provide images of the Sun’s poles.

49. Consider the following pairs of folk music with their associated state Folk Music State 1. Lavani : Gujrat 2. Rabindra Sangeet : West Bengal 3. Bhavageete : Karnataka 4. Naatupura Paadalgal : Tamil Nadu Which of the pairs given above is/are correctly matched? (a) 1, 2 and 3 only (b) 2, 3 and 4 only www.insightsonindia.com 39 Insights IAS INSTA 75 Days REVISION PLAN for Prelims 2020 - InstaTests

(c) 1, 3 and 4 only (d) 1, 2, 3 and 4

Solution: B

Lavani

• Lavani is a popular folk music of Maharashtra and was originally performed to entertain the soldiers. The song is usually performed by womenfolk and it conveys information pertaining to society and politics. Rabindra Sangeet

• Also known as Tagore Songs, Rabindra Sangeet is a set of folk songs, written and composed by eminent poet Rabindranath Tagore. Tagore wrote over 2,230 songs during his lifetime, all of which are sung during festivals and other rituals in West Bengal. Bhavageete

• Bhavageete is one of the most important folk music of Karnataka. Bhavageete literally means music of expression and hence, the singer’s expressions constitute the most important aspect of this form of music. Naatupura Paadalgal

• Naatupura Paadalgal is an ancient musical form of Tamil Nadu. Though folk music is fast disappearing in the state of Tamil Nadu due to the prominence given to Carnatic music, Naatupura Paadalgal remains quite important in the state.

50. Recently, JAGA mission received World Habitat Award, which was launched by (a) Haryana (b) Jharkhand (c) Odisha (d) West Bengal

Solution: C

www.insightsonindia.com 40 Insights IAS INSTA 75 Days REVISION PLAN for Prelims 2020 - InstaTests

• Odisha has won the ‘World Habitat Award’, global recognition for its ambitious initiative — Jaga Mission. • Recently, this project was also awarded the ‘India Geospatial Excellence Award’ for technological innovation in transforming the lives of urban poor. About Jaga mission:

• Odisha Liveable Habitat Mission “JAGA” is a society under Housing & Urban Development Department, Government of Odisha. • Composition: Headed by the Chief Secretary, Odisha as Chairman and Principal Secretary, H&UD as Member Secretary. Aims:

• Transform the slums into liveable habitat with all necessary civic infrastructure and services at par with the better off areas within the same urban local body (ULB). • Continuously improve the standard of the infrastructure and services and access to livelihood opportunities. • Leverage and converge various schemes/ programs/ funding opportunities by strengthening collaboration among various Departments and other Stakeholders. • Provide advisory support to Government of Odisha to examine options for policy reforms required for the sustainable transformation of lives of urban poor. World Habitat Award:

• This award is given by World Habitat, a UK-based organization, in partnership with United Nation (UN)-Habitat, every year, in recognition of innovative, outstanding, and revolutionary ideas, projects, and programmes from across the world.

DAY – 31 (InstaTest-31)

51. Consider the following statements regarding the Adaptation for Smallholder Agriculture Programme (ASAP): 1. It is Food and Agriculture Organization’s (FAO) flagship programme for channeling climate and environmental finance to smallholder farmers. 2. Women Empowerment is one of the focus areas of the Adaptation for Smallholder Agriculture Programme (ASAP) funds. Which of the statements given above is/are correct? (a) 1 only (b) 2 only (c) Both 1 and 2 (d) Neither 1 nor 2 www.insightsonindia.com 41 Insights IAS INSTA 75 Days REVISION PLAN for Prelims 2020 - InstaTests

Solution: B

The Adaptation for Smallholder Agriculture Programme (ASAP) is IFAD’s flagship programme for channelling climate and environmental finance to smallholder farmers. The programme is incorporated into IFAD’s regular investment processes and benefits from rigorous quality control and supervision systems.

• Thanks to the joint efforts and generous support of 10 donors, ASAP has received US$300 million in contributions. It has helped eight million vulnerable smallholders in 43 countries cope with the impact of climate change and build more resilient livelihoods. • In the future, ASAP will ensure that approaches for addressing climate-related risks are integrated into all of IFAD’s portfolio of loans and grants. • The ASAP fund allows IFAD country programmes to design projects from a climate- informed perspective and leverage resources for technical assistance. ASAP funds activities that focus on:

• policy engagement –supporting agricultural institutions in IFAD Member States seeking to achieve international climate change commitments and national adaptation priorities; • climate risk assessment – facilitating the systematic use of climate risk information when planning investments to increase resilience; • women’s empowerment – increasing the participation of women in, and their benefits from, climate-change adaptation activities; • private-sector engagement – strengthening the participation of the private sector and farmer groups in climate change adaptation and mitigation activities; • climate services – enhancing the use of climate information for when planning investments to increase resilience; • natural resource management and governance – strengthening the participation and ownership of smallholder farmers in decision-making processes; and improving technologies for the governance and management of climate-sensitive natural resources; • knowledge management – enhancing the documentation and dissemination of knowledge on approaches to climate-resilient agriculture. https://www.ifad.org/en/asap

52. Which of the following animals have been found on Pashupati Seal? 1. Elephant 2. Tiger 3. Rhinoceros 4. Bull

www.insightsonindia.com 42 Insights IAS INSTA 75 Days REVISION PLAN for Prelims 2020 - InstaTests

5. Antelopes Select the correct answer using the code given below: (a) 1, 2 and 3 only (b) 1, 2, 3 and 4 only (c) 1, 2, 3 and 5 only (d) 1, 2, 3, 4 and 5

Solution: C

• The most remarkable seal is the one depicted with a figure in the centre and animals around. This seal is generally identified as the Pashupati Seal by some scholars whereas some identify it as the female deity. This seal depicts a human figure seated cross legged. • An elephant and a tiger are depicted to the right side of the seated figure, while on the left a rhinoceros and a buffalo are seen. In addition to these animals two antelopes are shown below the seat. Seals such as these date from between 2500 and 1900 BCE and were found in considerable numbers in sites such as the ancient city of Mohenjodaro in the Indus Valley.

53. Arrange the following temple architecture chronologically from the earliest to latest 1. Brihadesvara temple 2. Kailasa temple 3. Sun temple at Konark 4. Ladkhan temple Select the correct answer using the code given below: (a) 2 4 1 3 (b) 4 2 1 3 (c) 4 2 3 1 (d) 2 4 3 1

Solution: B

www.insightsonindia.com 43 Insights IAS INSTA 75 Days REVISION PLAN for Prelims 2020 - InstaTests

The Ladkhan temple of Aihole belongs to about 5th century A.D. Here the architect has tried to give attention to the circumambulation path which is enclosed by means of a wall allowing devotees to have pradakshana or cirumambulatory of the holy of holies. The famous Kailasa temple at Ellora is in a class by itself because it is a rock-cut temple complex, which in many respects resembles the various rathas at Mahabalipuram. This temple was constructed during the reign of the Rashtrakuta King Krishna and belongs to the middle of the 8th century A.D. The Brihadesvara temple which was erected about 1000 A.D. is a contemporary of the Rajarani temple of Bhubaneswar. The great temple, the grandest achievement of the artistic and architectural genius of Orissa is the Sun temple at Konark which was constructed by the eastern Ganga ruler Narasimha Varmana, about 1250 A.D.

54. Consider the following statements 1. The BioCarbon Fund Initiative for Sustainable Forest Landscapes is administered by the United Nations Environment Program (UNEP) 2. New Development Bank (NDB) is one of the implementing agencies for Clean Technology Fund (CTF) investments 3. The Global Energy Efficiency and Renewable Energy Fund (GEEREF) is managed by World Bank. Which of the statements given above is/are correct? (a) 1 and 2 only (b) 3 only (c) 2 only (d) None

Solution: D

The BioCarbon Fund Initiative for Sustainable Forest Landscapes supports developing countries’ efforts to reduce emission through testing jurisdictional approaches that integrate reducing deforestation and degradation, sustainable forest management with the climate smart agricultural practices to green supply chains.

• The BioCarbon Fund concept was first revealed at the Katoomba Group meeting in March 2002, and was approved by the World Bank’s Board of Executive Directors in 2003. The Fund became operational in 2004. is administered by the World Bank.

www.insightsonindia.com 44 Insights IAS INSTA 75 Days REVISION PLAN for Prelims 2020 - InstaTests

The Clean Technology Fund (CTF), one of two multi-donor Trust Funds within the Climate Investment Funds (CIFs), promotes scaled-up financing for demonstration, deployment and transfer of low-carbon technologies with significant potential for long-term greenhouse gas emissions savings. • The World Bank is the Trustee and Administrating Unit of the CTF Trust Fund. • The World Bank Group, the African Development Bank, the Asian Development Bank, the European Development Bank, and the Inter-American Development Bank are the implementing agencies for CTF investments. The Global Energy Efficiency and Renewable Energy Fund (GEEREF) is a Public-Private Partnership (PPP) designed to maximise the private finance leveraged through public funds funded by the European Commission and managed by the European Investment Bank. GEEREF is structured as a fund of funds, and invests in private equity sub-funds that specialise in financing small and medium-sized project developers and enterprises (SMEs) to implement energy efficiency and renewable energy projects in developing countries and economies in transition.

55. Consider the following statements regarding Dance 1. Kerala is the home of the classical dance Kathakali. 2. Chakiarkoothu, Koodiyattam, Krishnattam and Ramanattam are the ritual performing arts of Kerala which have had a direct influence on Kathakali. 3. Kathakali derives its textual sanction from Balarama Bharatam and Hastalakshana Deepika. Which of the statements given above is/are correct? (a) 1 and 2 only (b) 2 and 3 only (c) 1 and 3 only (d) 1, 2 and 3

Solution: D

Kathakali Dance

• Kerala is the home of several traditional dance and dance – drama forms, the most notable being Kathakali. • Kathakali, as a dance form popular today, is considered to be of comparatively recent origin. However, it is an art which has evolved from many social and religious theatrical forms which existed in the southern region in ancient times. Chakiarkoothu, Koodiyattam, Krishnattam and Ramanattam are few of the ritual performing arts of

www.insightsonindia.com 45 Insights IAS INSTA 75 Days REVISION PLAN for Prelims 2020 - InstaTests

Kerala which have had a direct influence on Kathakali in its form and technique. Legend has it that the refusal of the Zamorin of Calicut to send his Krishnattam troupe to Travancore, so enraged the Raja of Kottarakkara, that he was inspired to compose the Ramanattam. • Kathakali is a blend of dance, music and acting and dramatizes stories, which are mostly adapted from the Indian epics. It is a stylised art form, the four aspects of abhinaya – angika, aharya,vachika, satvika and the nritta, and natya aspects are combined perfectly. The dancer expresses himself through codified hastamudras and facial expressions, closely following the verses(padams) that are sung. Kathakali derives its textual sanction from Balarama Bharatam and Hastalakshana Deepika. • Kalasams are pure dance sequences where the actor is at great liberty to express himself and display his skills. The leaps, quick turns, jumps and the rhythmic co- ordination make kalasams, a joy to watch. • A Kathakali performance begins with the kelikottu, calling the audience to attention followed by the todayam. It is a devotional number performed where one or two characters invoke the blessings of the gods. Kelikottu is the formal announcement of the performance done in the evening when drums and cymbals are played for a while in the courtyard. A pure nritta piece known as the purappadu comes as a sequel to this. Then the musicians and drummers hold the stage entertaining the audience with an exhibition of their skills in melappada. Tiranokku is the debut on the stage of all characters other than the pacha or minukku. Thereafter, the play or the particular scene of the chosen play begins.

56. Consider the following statements regarding Surajkund International Crafts Mela 1. It is the largest crafts fair in the world. 2. The Mela is organized by the Surajkund Mela Authority and Haryana Tourism. Which of the statements given above is/are correct? www.insightsonindia.com 46 Insights IAS INSTA 75 Days REVISION PLAN for Prelims 2020 - InstaTests

(a) 1 only (b) 2 only (c) Both 1 and 2 (d) Neither 1 nor 2

Solution: C

Surajkund International Crafts Mela

• It is unique as it showcases the richness and diversity of the handicrafts, handlooms and cultural fabric of India, & is the largest crafts fair in the world. • Held at Faridabad. • The Mela is organized by the Surajkund Mela Authority & Haryana Tourism in collaboration with Union Ministries of Tourism, Textiles, Culture and External Affairs. • Theme state: For the 34th Surajkund International Crafts Mela-2020, the state of Himachal Pradesh has been chosen to be the Theme State. • The main motto behind this festival is to promote handicrafts, handlooms with the aid of craftsmen invited from all over the country. So basically this craft festival is a platform for artists across the globe to showcase their culture and talent.

57. Consider the following statements regarding the different committees related to banking sector: 1. Ratan P. Watal committee was set up to make recommendations for the promotion of digital payments in the country. 2. P.J. Nayak committee was constituted by the Ministry of Finance to review Governance of Boards of Banks in India. 3. Urjit Patel Committee recommended the nominal anchor or the target for inflation should be set at 6 per cent with a band of +/- 2 per cent around it. Which of the statements given above is/are correct? (a) 1 only (b) 2 and 3 only (c) 1 and 2 only (d) 3 only

Solution: A

www.insightsonindia.com 47 Insights IAS INSTA 75 Days REVISION PLAN for Prelims 2020 - InstaTests

• The Committee on Digital Payments constituted by the Ministry of Finance, Department of Economic Affairs under the Chairmanship of Shri. Ratan P. Watal, Principal Advisor, NITI Aayog and former Finance Secretary to the Government of Indi • The Committee to Review Governance of Boards of Banks in India was constituted by the RBI Governor on 20th January, 2014 which was chaired by P.J.Nayak. • An expert committee headed by Urjit R. Patel was appointed to examine the current monetary policy framework of the Reserve Bank of India (RBI) has suggested that the apex bank should adopt the new CPI (consumer price index) as the measure of the nominal anchor for policy communication. • The committee felt that inflation should be the nominal anchor for the monetary policy framework. The nominal anchor or the target for inflation should be set at 4 per cent with a band of +/- 2 per cent around it.

58. Kharchi puja is organized in the state of (a) Uttarakhand (b) Himachal Pradesh (c) Sikkim (d) Tripura

Solution: D

‘Kharchi puja’ begins in tripura

• The annual “Kharchi Puja” and festival is meant to cleanse the sins of mortal souls. • Originally a Hindu tribals’ festivity, it is now observed by all communities and religions. • The festival features 14 deities – Shiva, Durga, Vishnu, Laxmi, Saraswati, Kartik, Ganesha, Brahma, Abadhi (God of water), Chandra, Ganga, Agni, Kamdev and Himadri (Himalaya). • In 2019 the Kharchi Puja mela was celebrated with the theme of ‘Nesha Mukta Tripura and Save Water’.

59. Consider the following pairs of theatre forms and the region they are performed: Theatre Forms Region 1. Bhand Pather : Kashmir 2. Nautanki : Uttar Pradesh 3. Bhavai : Rajasthan 4. Bhaona : Jharkhand Which of the pairs given above is/are correctly matched? www.insightsonindia.com 48 Insights IAS INSTA 75 Days REVISION PLAN for Prelims 2020 - InstaTests

(a) 1 and 2 only (b) 2 and 3 only (c) 3 and 4 only (d) 1 and 4 only

Solution: A

• Bhand Pather, the traditional theatre form of Kashmir, is a unique combination of dance, music and acting. Satire, wit and parody are preferred for inducing laughter. In this theatre form, music is provided with surnai, nagaara and dhol. Since the actors of Bhand Pather are mainly from the farming community, the impact of their way of living, ideals and sensitivity is discernible. • Nautanki is usually associated with Uttar Pradesh. The most popular centres of this traditional theatre form are Kanpur, Lucknow and Haathras. The meters used in the verses are: Doha, Chaubola, Chhappai, Behar-e-tabeel. There was a time when only men acted in Nautanki but nowadays, women have also started taking part in the performances. Among those remembered with reverence is Gulab Bai of Kanpur. She gave a new dimension to this old theatre form. • Bhavai is the traditional theatre form of Gujarat. The centers of this form are Kutch and Kathiawar. The instruments used in Bhavai are: bhungal, tabla, flute, pakhaawaj, rabaab, sarangi, manjeera, etc. In Bhavai, there is a rare synthesis of devotional and romantic sentiments. • is the traditional theatre form of Madhya Pradesh. The term Maach is used for the stage itself as also for the play. In this theatre form songs are given prominence in between the dialogues. The term for dialogue in this form is bol and rhyme in narration is termed vanag. The tunes of this theatre form are known as rangat. • Bhaona is a presentation of the Ankia Naat of Assam. In Bhaona cultural glimpses of Assam, Bengal Orissa, Mathura and Brindavan can be seen. The Sutradhaar, or narrator begins the story, first in Sanskrit and then in either Brajboli or Assamese.

60. Consider the following statements regarding the cultural sites of India, recently seen in news: 1. The historic town of Orchha was founded in the 16th century by the Bundela Rajput Chief, Rudra Pratap. 2. Phupgaon is situated in the vast meander of the river Purna. 3. Markandeshwar temples belong to the Nagara group of temples. Which of the statements given above is/are correct? (a) 1 and 2 only www.insightsonindia.com 49 Insights IAS INSTA 75 Days REVISION PLAN for Prelims 2020 - InstaTests

(b) 2 and 3 only (c) 1 and 3 only (d) 1, 2 and 3

Solution: D

• Madhya Pradesh’s famous Orchha town has now been included in the tentative list of world heritage sites by UNESCO. • The historic town of Orchha, nestled on the banks of river Betwa, was founded in the 16th century by the Bundela Rajput Chief, Rudra Pratap. Here, the river Betwa splits into seven channels, also called the Satdhara. Legend goes that this is in honour of the seven erstwhile Chiefs of Orchha. • The ancient town seems frozen in time, with its many monuments continuing to retain their original grandeur even to this day. http://www.mptourism.com/tourist-places/orchha-tourism.html

• The recent excavation carried out by Archaeological Survey of India (ASI) at Maharashtra’s Phupgaon has revealed evidence of an Iron Age settlement in the Vidarbha region. • The site is situated in the vast meander of the river Purna, a major tributary of Tapi, which used to be a perennial river, but at present is completely dried-up due to the dam construction in the upper stream. The site is situated about 20 m away from the river bed and its one-third portion has been subjected to frequent erosion during the heavy water current in the earlier times. https://pib.gov.in/newsite/PrintRelease.aspx?relid=192753

• The restoration work of Markandeshwar temple in Maharashtra by the Archaeological Survey of Indiais in full swing. Known as the “Khajuraho of Vidarbha”,the temple of Markandadeo is situated on the bank of River Wainganga in district Gadchiroli of Maharashtra. • The temples belong to the Nagara group of temples of North India. On stylistic grounds, their date ranges in between 9-12th centuries CE. The temples belong to saiva, vaishnava and sakta faith. Most of the temples have a simple plan, with ardhamandapa, mandapa, antarala and garbhagriha forming the component of the entire set up. https://pib.gov.in/newsite/PrintRelease.aspx?relid=191808

www.insightsonindia.com 50 Insights IAS INSTA 75 Days REVISION PLAN for Prelims 2020 - InstaTests

61. Consider the following statements regarding puppetry 1. The earliest reference to the art of puppetry is found in Tamil classic ‘Silappadikaaram’ written around the 1st or 2nd century B.C. 2. Natyashastra, the masterly treatise on dramaturgy does not refer to the art of puppetry. Which of the statements given above is/are correct? (a) 1 only (b) 2 only (c) Both 1 and 2 (d) Neither 1 nor 2

Solution: C

• Ancient Hindu philosophers have paid the greatest tribute to puppeteers. They have likened God Almighty to a puppeteer and the entire universe to a puppet stage. Srimad Bhagavata, the great epic depicting the story of Lord Krishna in his childhood say that with three strings-Satta, Raja and Tama, the God manipulates each object in the universe as a marionette. • In Sanskrit terminology Puttalika and Puttika means ‘little sons’. The root of Puppet is derived from the latin word ‘Pupa’ meaning a doll. India is said to be the home of puppets, but it is yet to awaken to its unlimited possibilities. The earliest reference to the art of puppetry is found in Tamil classic ‘Silappadikaaram’ written around the 1st or 2nd century B.C. • Natyashastra, the masterly treatise on dramaturgy written sometime during 2nd century BC to 2nd century AD., does not refer to the art of puppetry but the producer- cum-director of the human theatre has been termed as ‘Sutradhar’ meaning the holder of strings. The word might have found its place in theatre-terminology long before Natyashastra was written but it must come from marionette theatre. Puppetry, therefore, must have originated in India more than 500 years before Christ.

62. Consider the following statements regarding Buddhist and Jain literature 1. The Buddhist canonical literature was written in Pali language. 2. Many of the Jain tales were written in some forms of Prakrit. 3. Both Buddhism and Jainism have refrained from using Sanskrit in any form. Which of the statements given above is/are correct? (a) 1 and 2 only (b) 2 and 3 only www.insightsonindia.com 51 Insights IAS INSTA 75 Days REVISION PLAN for Prelims 2020 - InstaTests

(c) 1 and 3 only (d) 1, 2 and 3

Solution: A

Literature in Pali and Prakrit

• Pali and Prakrit were the spoken languages of Indians after the Vedic period. Prakrit in the widest sense of the term, was indicative of any language that in any manner deviated from the standard one, i.e. Sanskrit. Pali is archaic Prakrit. In fact, Pali is a combination of various dialects. These were adopted by Buddhist and Jain sects in ancient India as their sacred languages. • Lord Buddha (500 B.C.) used Pali to give his sermons. All the Buddhist canonical literature is in Pali which includes Tipitaka (threefold basket). The first basket, Vinaya Pitaka, contains the monastic rules of the Order of Buddhist monks. The second basket, Sutta Pitaka, is the collection of the speeches and dialogues of the Buddha. The third basket, the Abhidhamma Pitaka, elucidates the various topics dealing with ethics, psychology or theory of knowledge. The jataka Kathas are non-canonical Buddhist literature in which stories relating to the former births of the Buddha (Bodhi- sattva or the would-be Buddha) are narrated. These stories propagate Buddhist religious doctrines and are available in both Sanskrit and Pali. As the jataka tales grew in bulk, they assimilated popular tales, ancient mythology, stories from older religious traditions, etc. Jatakas are, in fact, based on the common heritage of the Indian masses. Buddhist literature is also abundantly available in Sanskrit, which includes the great epic Buddhacharita by Aswaghosha (78 A.D.). • Like the Buddhist stories, the Jain tales in general are didactic in character. They are written in some forms of Prakrit. The word Jain is derived from the root ji (to conquer) and signifies the religion of those who have conquered the lust for life. Jain canonical literature by Jain saints, as well as a large number of works on lexicography and grammar by Hemachandra (1088 A.D.-?), is well known. Much also in the way of moral tale and poetry are to be found. However, Prakrit is well known for Gathasaptashati (700 verses) by Hala (300 A.D.), the best example of erotic literature. It is a compilation of 700 verses along with his own contribution of 44 poems. It is interesting to note that quite a few poetesses like Pahai, Mahavi, Reva, Roha and Sasippaha are included in the anthology. The vast Katha (story) literature of Prakrit, written with a conspicuous religious overtone, even by Jain saints, is full of erotic elements. The author of the Vasudevahindi ascribes this changed approach of the Jain authors to the fact that it is easy to teach religion cloaked by erotic episodes, like sugar-coated medicine. The characteristic of Prakrit poetry is its subtlety; the inner meaning (Hiyaali) is its soul. Jain literature is available in Sanskrit too, like the Upamitibhava Prapancha Katha of Siddharasi (906 A.D.).

www.insightsonindia.com 52 Insights IAS INSTA 75 Days REVISION PLAN for Prelims 2020 - InstaTests

63. Consider the following statements regarding Gram Nyayalayas 1. The Gram Nyayalaya shall be bound by the rules of evidence provided in the Indian Evidence Act, 1872 2. They have jurisdiction over only civil offences 3. The pecuniary jurisdiction of the Nyayalayas are fixed by the respective High Courts. Which of the statements given above is/are correct? (a) 3 only (b) 1 and 3 only (c) 2 and 3 only (d) 1, 2 and 3

Solution: A

• The Gram Nyayalayas Act, 2008 has been enacted to provide for the establishment of the Gram Nyayalayas at the grass roots level for the purposes of providing access to justice to the citizens at their doorsteps and to ensure that opportunities for securing justice are not denied to any citizen due to social, economic or other disabilities. • The Gram Nyayalaya shall not be bound by the rules of evidence provided in the Indian Evidence Act, 1872 but shall be guided by the principles of natural justice and subject to any rule made by the High Court. Jurisdiction:

• A Gram Nyayalaya have jurisdiction over an area specified by a notification by the State Government in consultation with the respective High Court. • The Court can function as a mobile court at any place within the jurisdiction of such Gram Nyayalaya, after giving wide publicity to that regards. • They have both civil and criminal jurisdiction over the offences. • The pecuniary jurisdiction of the Nyayalayas are fixed by the respective High Courts. • Gram Nyayalayas has been given power to accept certain evidences which would otherwise not be acceptable under Indian Evidence Act. Significance:

• The setting up of Gram Nyayalayas is considered as an important measure to reduce arrears and is a part of the judicial reforms. It is estimated that Gram Nyayalayas can reduce around 50% of the pendency of cases in subordinate courts and can take care of the new litigations which will be disposed within six months.

www.insightsonindia.com 53 Insights IAS INSTA 75 Days REVISION PLAN for Prelims 2020 - InstaTests

64. Consider the following pairs of folk music and the States they are originally performed Folk Music State 1. Geet : Rajasthan 2. Pankhida : Uttar Pradesh 3. Pandavani : Chhattisgarh 4. Mando : Goa Which of the pairs given above is/are correctly matched? (a) 1 and 2 only (b) 2 and 3 only (c) 3 and 4 only (d) 1 and 4 only

Solution: C

Rasiya Geet, Uttar Pradesh

• The rich tradition of singing Rasiya Geet flourished in Braj which is the sacred land of Lord Krishna’s charming leelas from time immemorial. This is not confined to any particular festival, but is closely woven into the very fabric of daily life and day to day chores of its people. ‘Rasiya’ word is derived from the word rasa (emotion) because rasiya means that which is filled with rasa or emotion. It reflects the personality of the singer as well as the nature of the song. Pankhida, Rajasthan

• Sung by the peasants of Rajasthan while doing work in the fields, the peasants sing and speak while playing algoza and manjira. The literal meaning of the word ‘Pankhida’ is lover. Lotia, Rajasthan

• ‘Lotia’ is sung in the chaitra month during the festival – ‘Lotia’. Women bring lotas (a vessel to fill water) and kalash (a vessel considered to be auspicious to fill water during worship) filled with water from ponds and wells. They decorate them with flowers and come home. Pandavani, Chhattisgarh

• In Pandavani, tales from Mahabharata are sung as a ballad and one or two episodes are chosen for the night’s performance. The main singer continuously sits throughout the performance and with powerful singing and symbolic gestures he assumes all the characters of the episode one after another.

www.insightsonindia.com 54 Insights IAS INSTA 75 Days REVISION PLAN for Prelims 2020 - InstaTests

Shakunakhar – Mangalgeet, Kumaon

• Numberless songs are sung on auspicious occasions in the foothills of Himalaya. Shakunakhar are sung during religious ceremonies of baby-shower, child-birth, Chhati (a ritual done on the sixth day from the birth of a child) Ganesh pooja etc. These songs are sung by only ladies, without any instrument. Barhamasa, Kumaon

• This regional music from Kumaon is describing the twelve months of an year, each with its specific qualities. In one of the songs the Ghughuti bird symbolizes the onset of chait month. A girl in her in laws place asks this bird not to speak because she is disturbed with the memories of her mother (Ija) and she is feeling sad. Mando, Goa

• Goan regional music is a treasury of the traditional music of the Indian subcontinent. Mando, the finest creation of Goan song is a slow verse and refrain composition dealing with love, tragedy and both social injustice and political resistance during Portuguese presence in Goa.

65. Consider the following statements regarding National School of Drama 1. It was set up by the Sangeet Natak Akademi as one of its constituent units. 2. Bharat Rang Mahotsav is the annual theatre festival of National School of Drama (NSD). Which of the statements given above is/are correct? (a) 1 only (b) 2 only (c) Both 1 and 2 (d) Neither 1 nor 2

Solution: C

National School of Drama

• Set up by the Sangeet Natak Akademi as one of its constituent units in 1959. • In 1975, it became an independent entity and was registered as an autonomous organization under the Societies Registration Act XXI of 1860, fully financed by the Ministry of Culture, Government of India.

www.insightsonindia.com 55 Insights IAS INSTA 75 Days REVISION PLAN for Prelims 2020 - InstaTests

• Bharat Rang Mahotsav, or the ‘National Theatre Festival’, established in 1999, is the annual theatre festival of National School of Drama (NSD), held in New Delhi, today it is acknowledged as the largest theatre festival of Asia, dedicated solely to theatre.

66. Consider the following statements regarding Payment and settlement systems Act, (PSS) 2007 1. It provides for the regulation and supervision of payment systems in India 2. Any person desirous of commencing or operating a payment system needs to apply for authorization under the PSS Act, 2007 Which of the statements given above is/are correct? (a) 1 only (b) 2 only (c) Both 1 and 2 (d) Neither 1 nor 2

Solution: C

• The PSS Act, 2007 provides for the regulation and supervision of payment systems in India and designates the Reserve Bank of India (Reserve Bank) as the authority for that purpose and all related matters. • The Reserve Bank is authorized under the Act to constitute a Committee of its Central Board known as the Board for Regulation and Supervision of Payment and Settlement Systems (BPSS), to exercise its powers and perform its functions and discharge its duties under this statute. • In terms of Section 4 of the PSS Act, 2007 no person other than the Reserve Bank can operate or commence a payment system unless authorized by the Reserve Bank. Any person desirous of commencing or operating a payment system needs to apply for authorization under the PSS Act, 2007(Section 5). https://m.rbi.org.in/Scripts/FAQView.aspx?Id=73

67. Consider the following statements regarding Amaravati School of Art: 1. It was developed indigenously and not influenced by external cultures. 2. The sculptures were made using spotted red sandstone. 3. It was patronized by Kushana rulers. Which of the statements given above is/are correct?

www.insightsonindia.com 56 Insights IAS INSTA 75 Days REVISION PLAN for Prelims 2020 - InstaTests

(a) 1 only (b) 2 and 3 only (c) 1 and 3 only (d) 1, 2 and 3

Solution: A

Mathura School

• It was developed indigenously and not influenced by external cultures. • The sculptures of Mathura School were made using spotted red sandstone. • Influence of all three religions of the time, i.e. Hinduism, Jainism and Buddhism. • Patronised by Kushana rulers. • Buddha is shown in delighted mood with a smiling face. The body symbolizes mascularity, wearing tight dress. The face and head are shaven. Buddha is seated in padmasana with different mudras and his face reflects grace. A similar protuberance is shown on the head. Amaravati School

• It was developed indigenously and not influenced by external cultures. • The sculptures of Amaravati School were made using white marbles. • Mainly Buddhist influence. • Patronised by Satvahana rulers. • Since the sculptures are generally part of a narrative art, there is less emphasis on the individual features of Buddha. The sculptures generally depict life stories of Buddha and the Jataka tales i.e., previous lives of Buddha in both human and animal form.

www.insightsonindia.com 57 Insights IAS INSTA 75 Days REVISION PLAN for Prelims 2020 - InstaTests

68. Which of the following are the features of Indo-Islamic architecture 1. The human and animal figures were used extensively. 2. The buildings had intricate jaali works, which signifies the importance of light in Islamic religion. 3. Indo-Islamic architecture used calligraphy as means of decoration. Which of the statements given above is/are correct? (a) 1 only (b) 2 and 3 only (c) 1 and 3 only (d) 1, 2 and 3

Solution: B

The features of Indo-Islamic architecture are

• The Indo-Islamic architecture avoided the use of human and animal figures. • While the Hindu architecture had become too congested, the Indo-Islamic architecture introduced spaciousness, massiveness and breadth to it. • While the previous structures used sculptures as means of decoration, the Indo- Islamic architecture used calligraphy as means of decoration. • Arabesque method was also used for decoration. Arabesque meant the use of geometrical vegetal ornamentation and was characterised by a continuous stem which split regularly, producing a series of counterpoised, leafy, secondary stems. The secondary stems, in turn, split again or returned to merge with the main stem creating a decorative pattern. • The buildings had intricate jaali works, which signifies the importance of light in Islamic religion. • The Islamic rulers introduced the Charbagh style of gardening, in which a square block was divided into four adjacent identical gardens. • The architectures of these times also used the pietra-dura technique for the inlay of precious stones and gems into the stone walls. • Another unique feature was the use of foreshortening technique in the buildings, such that the inscriptions appear to be closer than it really is.

www.insightsonindia.com 58 Insights IAS INSTA 75 Days REVISION PLAN for Prelims 2020 - InstaTests

69. Consider the following statements regarding Nyaya Bandhu (Mobile Application) scheme 1. It provides a platform to connect the registered eligible beneficiaries with the practicing advocates who are willing to volunteer their time and money in providing their services. 2. It aims to fulfill the Constitutional commitment under Article 49 for providing access to justice to all. Which of the statements given above is/are correct? (a) 1 only (b) 2 only (c) Both 1 and 2 (d) Neither 1 nor 2

Solution: A

• Nyaya Bandhu scheme under Ministry of Law aims to provide a platform to connect the registered eligible beneficiaries with the practicing advocates who are willing to volunteer their time and money in providing their services as a pro bono in pursuance of Constitutional commitment under Article 39A https://pib.gov.in/newsite/PrintRelease.aspx?relid=188736

www.insightsonindia.com 59 Insights IAS INSTA 75 Days REVISION PLAN for Prelims 2020 - InstaTests

70. Consider the following statements regarding Sittanavasal Cave Paintings 1. Sittanavasal Caves are located in the state of Tamil Nadu 2. These famous rock-cut caves are known for the paintings in the Buddhist structures. Which of the statements given above is/are correct? (a) 1 only (b) 2 only (c) Both 1 and 2 (d) Neither 1 nor 2

Solution: A

Sittanavasal Cave (Arivar Koil) Paintings

• Located 16 km northwest of Pudukkottai town in Tamil Nadu, these famous rock-cut caves are known for the paintings in the Jain temples. These murals have close resemblance to Bagh and Ajanta paintings. The paintings are not only on the walls but also on the ceiling and pillars. The paintings are with the theme of Jain Samavasarana (Preaching hall). • Some scholars believe that these caves belong to the Pallava period, when king Mahendravarman I excavated the temple, while the others attribute them to when Pandya ruler renovated the shrine in the 7th century. The medium used for painting is vegetable and mineral dyes, and is done by putting colours on surface of thin wet lime plaster. The common colours include yellow, green, orange, blue, black and white. • The central element of the paintings in Sittanavasal is a pond with lotuses. Flowers in this pond are collected by monks, there are shown ducks, swans, fishes and animals. This scene shows Samavasarana – important scene in Jain religion. Samava-sarana is special, beautiful audience hall where Tirthankaras delivered sermons after they reached realisation (kevala-gnana). Bulls, elephants, apsaras and gods gathered in this audience hall to witness this grand scene.

71. Consider the following statements regarding National Productivity Council (NPC) 1. It is an autonomous registered society under Ministry of Human Resource and Development. 2. It aims to improve productivity of educational institutes in rural India 3. It is a constituent of the Tokyo-based Asian Productivity Organization (APO) Which of the statements given above is/are correct?

www.insightsonindia.com 60 Insights IAS INSTA 75 Days REVISION PLAN for Prelims 2020 - InstaTests

(a) 1 and 2 only (b) 2 only (c) 3 only (d) 1, 2 and 3

Solution: C

National Productivity Council (NPC) aims to promote the cause of productivity in all sectors of the Indian economy.

• Established by the Ministry of Industry, Government of India in 1958, it is an autonomous, multipartite, non-profit organization and has been registered as a Society under the Societies Registration Act XXI of 1860. • NPC is a constituent of the Tokyo-based Asian Productivity Organisation (APO), an Inter-Governmental Body, of which the Government of India is a founder member. https://www.npcindia.gov.in/NPC/User/about

72. Consider the following statements regarding the characteristics of Carnatic Music 1. There is a vast freedom to improvise. 2. There are several sub-styles of singing. 3. It gives more emphasis on vocal music. Which of the statements given above is/are correct? (a) 1 only (b) 2 and 3 only (c) 3 only (d) 1, 2 and 3

Solution: C

Carnatic Music

• No freedom to improvise. • Only one particular prescribed style of singing. • More emphasis on vocal music. • Doesn’t adhere to any time. www.insightsonindia.com 61 Insights IAS INSTA 75 Days REVISION PLAN for Prelims 2020 - InstaTests

• 72 ragas. • Veena, Mrindangum and Mandolin. • Usually South India. • Flute and Violin

73. Consider the following statements regarding Agri-Market Infrastructure Fund (AMIF) 1. It will be created under the ambit of Reserve Bank of India 2. It will provide the state/ UT governments subsidized loans for their proposal for developing marketing infrastructure Which of the statements given above is/are correct? (a) 1 only (b) 2 only (c) Both 1 and 2 (d) Neither 1 nor 2

Solution: B

Agri-Market Infrastructure Fund (AMIF):

• The fund will be created with NABARD for development and up-gradation of agricultural marketing infrastructure in Gramin Agricultural Markets and Regulated Wholesale Markets. • AMIF will provide the State/UT Governments subsidized loan for their proposal for developing marketing infrastructure in 585 Agriculture Produce Market Committees (APMCs) and 10,000 Grameen Agricultural Markets (GrAMs). • States may also access AMIF for innovative integrated market infrastructure projects including Hub and Spoke mode and in Public Private Partnership mode. https://www.insightsonindia.com/2019/02/07/agri-market-infrastructure-fund-amif/

74. Consider the following statements regarding Nyaya School of philosophy: 1. They believe in the technique of logical thinking to achieve salvation. 2. On the concept of God, they argue that the creation of the Universe was through God’s hands. Which of the statements given above is/are correct? (a) 1 only

www.insightsonindia.com 62 Insights IAS INSTA 75 Days REVISION PLAN for Prelims 2020 - InstaTests

(b) 2 only (c) Both 1 and 2 (d) Neither 1 nor 2

Solution: C

Nyaya School

• As the name of the school suggests, they believe in the technique of logical thinking to achieve salvation. They consider the life, death and salvation to be like mysteries that can be solved through logical and analytical thinking. Furthermore, they argue that acquiring ‘real knowledge’ can only accrue salvation. This school of thought was supposed to be founded by Gautama who is also identified as the author of the Nyaya Sutra. • They argue that by using logical tools like inference, hearing and analogy; a human being could verify the truth of a proposition or statement. • On the concept of God, they argue that the creation of the Universe was through God’s hands. They also believe that God not only created the Universe but also sustained and destroyed it. This philosophy constantly stressed on systematic reasoning and thinking.

75. Tungareshwar Wildlife Sanctuary, sometime seen in the news, is located in which of the following state? (a) Maharashtra (b) Tamil Nadu (c) Gujarat (d) Madhya Pradesh

Solution: A

Tungareshwar wildlife sanctuary also known as Tungareshwar National Park is located at Vasai in Palghar in Maharashtra. The sanctuary is over 85 km2 and forms a corridor between Sanjay Gandhi National Park and Tansa wildlife sanctuary. Tungareshwar was declared a wildlife sanctuary in 2003.

www.insightsonindia.com 63 Insights IAS INSTA 75 Days REVISION PLAN for Prelims 2020 - InstaTests https://www.thehindu.com/sci-tech/energy-and-environment/bullet-train-gets-green-light- via-flamingo-haven-national-park/article26188009.ece

DAY – 32 (InstaTest-32)

76. Consider the following statements regarding SnowEx campaign 1. It aims to create awareness regarding the declining number of Snow Leopard species in Himalayan region. 2. It was launched by The World Wide Fund for Nature (WWF) Which of the statements given above is/are correct? (a) 1 only (b) 2 only (c) Both 1 and 2 (d) Neither 1 nor 2

Solution: D

• NASA has launched a SnowEx seasonal campaign; it focuses on airborne campaigns and field work, and on comparing the various sensing technologies, from the mature to the more experimental, in globally-representative types of snow. • It aims to address the most important gaps in snow remote sensing knowledge and thus lay the groundwork for a future snow satellite mission. It is a five year program.

77. Consider the following pairs of organizations and their founders Organization Founder 1. The Bangabhasha : Ishwar Chandra Prakasika Sabha Vidyasagar 2. The East India : Dadabhai Naoroji Association 3. The Indian League : Sisir Kumar Ghosh Which of the pairs given above is/are correctly matched? (a) 1 only (b) 2 and 3 only (c) 1 and 3 only

www.insightsonindia.com 64 Insights IAS INSTA 75 Days REVISION PLAN for Prelims 2020 - InstaTests

(d) 1, 2 and 3

Solution: B

• The Bangabhasha Prakasika Sabha was formed in 1836 by associates of Raja Rammohan Roy. • The East India Association was organised by Dadabhai Naoroji in 1866 in London to discuss the Indian question and influence public men in England to promote Indian welfare. Later, branches of the association were started in prominent Indian cities. • The Indian League was started in 1875 by Sisir Kumar Ghosh with the object of “stimulating the sense of nationalism amongst the people” and of encouraging political education. • The Indian Association of Calcutta (also known as the Indian National Association) superseded the Indian League and was founded in 1876 by younger nationalists of Bengal led by Surendranath Banerjea and Ananda Mohan Bose.

78. Which of the following was/were discontent among Sepoys during British rule? 1. The discontent of the sepoys was limited to military matters. 2. The passing of General Service Enlistment Act and not giving the foreign service allowance (bhatta). 3. The conditions of service in the Company’s army came into conflict with the religious beliefs and prejudices of the sepoys. 4. The Indian sepoy was discriminated against racially and in matters of promotion and privileges. Select the correct answer using the code given below: (a) 1, 2 and 3 only (b) 2, 3 and 4 only (c) 1, 3 and 4 only (d) 1, 2, 3 and 4

Solution: B

Discontent Among Sepoys

• The conditions of service in the Company’s Army and cantonments increasingly came into conflict with the religious beliefs and prejudices of the sepoys. Restrictions

www.insightsonindia.com 65 Insights IAS INSTA 75 Days REVISION PLAN for Prelims 2020 - InstaTests

on wearing caste and sectarian marks and secret rumours of proselytizing activities of the chaplains (often maintained on the Company’s expenses which meant at Indian expense) were interpreted by Indian sepoys, who were generally conservative by nature, as interference in their religious affairs. • To the religious Hindu of the time, crossing the seas meant loss of caste. In 1856, Lord Canning’s government passed the General Service Enlistment Act which decreed that all future recruits to the Bengal Army would have to give an undertaking to serve anywhere their services might be required by the government. This caused resentment. The Indian sepoy was equally unhappy with his emoluments compared to his British counterpart. A more immediate cause of the sepoys’ dissatisfaction was the order that they would not be given the foreign service allowance (bhatta) when serving in Sindh or in Punjab. The annexation of Awadh, home of many of the sepoys, further inflamed their feelings. • The Indian sepoy was made to feel a subordinate at every step and was discriminated against racially and in matters of promotion and privileges. The discontent of the sepoys was not limited to military matters.

79. Consider the following statements regarding 1000 springs initiative 1. It aims to improve access to safe and adequate water for the tribal communities living in difficult and inaccessible part of rural areas in the country. 2. It includes the provision of water for irrigation 3. It was jointly launched by Ministry of Tribal Affairs and Ministry of Jal Shakti Which of the statements given above is/are correct? (a) 2 only (b) 1 and 3 only (c) 1 and 2 only (d) 1, 2 and 3

Solution: C

About 1000 spring’s initiative:

• Union Tribal Affairs Ministry has launched “1000 Spring Initiatives” and an online portal on GIS-based Spring Atlas with hydrological and chemical properties of the Springs on the occasion. • Aim: To improve access to safe and adequate water for the tribal communities living in difficult and inaccessible part of rural areas in the country. It is an integrated solution around natural springs.

www.insightsonindia.com 66 Insights IAS INSTA 75 Days REVISION PLAN for Prelims 2020 - InstaTests

Key features:

• It includes the provision of infrastructure for piped water supply for drinking; provision of water for irrigation; community-led total sanitation initiatives; and provision for water for backyard nutrition gardens, generating sustainable livelihood opportunities for the tribal people. Implementation:

• Under this initiative, more than 70 young tribal youths from the rural belt of three districts of Odisha namely, Kalahandi, Khandamal and Gajapati have been trained as barefoot hydro geologists by combining traditional and scientific knowledge for identification and mapping of springs, and undertaking rejuvenation and protection measures in their habitations.

80. Consider the following statements regarding Doctrine of Lapse 1. Lord Dalhousie was the originator of the concept of Doctrine of Lapse. 2. The first state to be annexed under Dalhousie’s Doctrine of Lapse was Jhansi. 3. Awadh was annexed on the grounds of misgovernment. Which of the statements given above is/are correct? (a) 3 only (b) 1 and 3 only (c) 1 and 2 only (d) 2 and 3 only

Solution: A

Doctrine of Lapse

• In simple terms, the doctrine stated that the adopted son could be the heir to his foster father’s private property, but not the state; it was for the paramount power (the British) to decide whether to bestow the state on the adopted son or to annex it. • Though this policy is attributed to Lord Dalhousie (1848-56), he was not its originator. It was a coincidence that during his governor-generalship several important cases arose in which the ‘Doctrine’ could be applied. Dalhousie showed too much zeal in enforcing this policy which had been theoretically enunciated on some previous occasions. His predecessors had acted on the general principle of avoiding annexation if it could be avoided; Dalhousie in turn acted on the general principle of annexing if he could do so legitimately.

www.insightsonindia.com 67 Insights IAS INSTA 75 Days REVISION PLAN for Prelims 2020 - InstaTests

Annexed Lapsed States

• It was a matter of chance that during Lord Dalhousie’s term many rulers of states died without a male issue and seven states were annexed under the Doctrine of Lapse. The most important of these were Satara (1848), Jhansi and Nagpur (1854). The other small states included Jaitpur (Bundelkhand), Sambhalpur (Orissa), and Baghat (Madhya Pradesh). Lord Dalhousie annexed Awadh in 1856 after deposing Nawab Wajid Ali Shah on grounds of misgovernment. Many states were annexed before Satara 1. Jaintia 1803 2. Kozhikode 1806 3. Guler 1813 4. Kannanur 1819 5. Kittur 1824

81. Consider the following statements regarding Paika Rebellion 1. The Paiks of Odisha were the traditional landed militia and enjoyed rent free land tenures for their military service and policing functions on a hereditary basis. 2. The rebellion was organized under the leadership of Bakshi Jagabandhu Bidyadhar. Which of the statements given above is/are correct? (a) 1 only (b) 2 only (c) Both 1 and 2 (d) Neither 1 nor 2

Solution: C

Paika Rebellion (1817)

• The Paiks of Odisha were the traditional landed militia (‘foot soldiers’ literally) and enjoyed rent free land tenures for their military service and policing functions on a hereditary basis. • The English Company’s conquest of Odisha in 1803, and the dethronement of the Raja of Khurda had greatly reduced the power and prestige of the Paiks. • Bakshi Jagabandhu Bidyadhar had been the military chief of the forces of the Raja of Khurda. In 1814, Jagabandhu’s ancestral estate of Killa Rorang was taken over by the Company, reducing him to penury.

www.insightsonindia.com 68 Insights IAS INSTA 75 Days REVISION PLAN for Prelims 2020 - InstaTests

• With active support of Mukunda Deva, the last Raja of Khurda, and other zamindars of the region, Bakshi Jagabandhu Bidyadhar led a sundry army of Paikas forcing the East India Company forces to retreat for a time. • Jagabandhu, declared an outlaw, along with other rebels, was sheltered by the Raja of Nayagarh. Although Dinabandhu Santra and his group surrendered in November 1818, Jagabandhu evaded British vigilance. • The Paika rebels resorted to guerilla tactics. The rebellion was brutally repressed by 1818. Priests at the Puri temple who had sheltered Jagabandhu were caught and hanged. Paiks on the whole suffered greatly. In 1825 Jagabandhu surrendered under negotiated terms. (Some sources say he was captured and died in captivity in 1829). • The Paik Rebellion succeeded in getting large remissions of arrears, reductions in assessments, suspension of the sale of the estates of defaulters at discretion, a new settlement on fixed tenures and other adjuncts of a liberal governance.

82. Consider the following statements regarding the Rashtriya Vayoshri Yojana 1. It is a scheme for providing Physical Aids and Assisted-living Devices for Senior citizens belonging to BPL category. 2. The scheme is fully funded by the Central Government. 3. Under the scheme, the physical aids will be provided only to the senior citizens of the nation. Which of the statements given above is/are correct? (a) 1 and 3 only (b) 2 and 3 only (c) 1 only (d) 1, 2 and 3

Solution: D

Rashtriya Vayoshri Yojana:

• Coverage: Senior Citizens, belonging to BPL category and suffering from any of the age-related disability/infirmity Low vision, Hearing impairment, Loss of teeth and Locomotor disability. • What assistance is provided? Assisted-living devices which can restore near normalcy in their bodily functions, overcoming the disability/infirmity manifested. They are free-of cost.

www.insightsonindia.com 69 Insights IAS INSTA 75 Days REVISION PLAN for Prelims 2020 - InstaTests

• Funding: This is a Central Sector Scheme, fully funded by the Central Government. The expenditure for implementation of the scheme will be met from the “Senior Citizens’ Welfare Fund “. • Under the scheme, the physical aids will be provided only to the senior citizens of the nation. Other key facts:

• In case of multiple disabilities/infirmities manifested in the same person, the assistive devices will be given in respect of each disability/impairment. • Beneficiaries in each district will be identified by the State Governments/UT Administrations through a Committee chaired by the Deputy Commissioner/District Collector. • As far as possible, 30% of the beneficiaries in each district shall be women.

83. Black Hole Tragedy is associated with (a) The Carnatic wars (b) The Anglo Maratha wars (c) The Battle of Buxar (d) The Battle of Plassey

Solution: D

• Black Hole Tragedy is associated with The Battle of Plassey. • Siraj-ud-daula is believed to have imprisoned 146 English persons who were lodged in a very tiny room due to which 123 of them died of suffocation. However, historians either do not believe this story, or say that the number of victims must have been much smaller.

84. Which of the following were the items of export during Mughal periods? 1. Raw silk and silk fabrics 2. Saltpetre 3. Pepper 4. Coffee Select the correct answer using the code given below: (a) 1, 2 and 3 only (b) 2, 3 and 4 only www.insightsonindia.com 70 Insights IAS INSTA 75 Days REVISION PLAN for Prelims 2020 - InstaTests

(c) 1, 3 and 4 only (d) 1, 2, 3 and 4

Solution: A

India’s trade during Mughal period: 1. Items of Import

• From Persian Gulf Region— pearls, raw silk, wool, dates, dried fruits, and rose water; • From Arabia—coffee, gold, drugs, and honey; • from China—tea, sugar, porcelain, and silk; • from Tibet—gold, musk, and woollen cloth; • from Africa—ivory and drugs; • from Europe—woollen cloth, copper, iron, lead and paper. 2. Items of Export: Cotton textiles, raw silk and silk fabrics, hardware, indigo, saltpetre, opium, rice, wheat, sugar, pepper and other spices, precious stones, and drugs.

85. Consider the following statements regarding National Calamity Contingent Duty 1. It was introduced under the Disaster Management Act 2005. 2. It is tax imposed on pan masala, cigarettes, cellular phones and other tobacco products. 3. NCCD in the present form has been imposed as a surcharge on Excise Duty Which of the statements given above is/are correct? (a) 1 only (b) 2 and 3 only (c) 3 only (d) 1, 2 and 3

Solution: B

National Calamity Contingent Duty:

• The NCCD is tax imposed on pan masala, cigarettes, cellular phones and other tobacco products. • It was introduced under Finance Act, 2001.

www.insightsonindia.com 71 Insights IAS INSTA 75 Days REVISION PLAN for Prelims 2020 - InstaTests

• NCCD in the present form has been imposed as a surcharge on Excise Duty. Section 136(1) of the Finance Act, 2001 states that there shall be levied and collected for the purposes of the Union, by surcharge, a duty of excise, to be called the National Calamity Contingent duty. • When the goods are covered under GST, and Excise duty is not levied on those goods; no surcharge can be levied. This no NCCD can be levied on tobacco and tobacco products, which are covered under GST regime.

86. Consider the following statements regarding regional kingdom of Hyderabad 1. The founder of the regional kingdom of Hyderabad was Kilich Khan. 2. It was Nizam-ul-Mulk who had first conceived the idea of an independent state in the Deccan. Which of the statements given above is/are correct? (a) 1 only (b) 2 only (c) Both 1 and 2 (d) Neither 1 nor 2

Solution: A

• The founder of the Asaf-Jah house of Hyderabad was Kilich Khan, popularly known as Nizam-ul-Mulk. It was Zulfikar Khan who had first conceived the idea of an independent state in the Deccan. But with his death in 1713 the dream remained unfulfilled. Kilich Khan, disgusted with the Mughal emperor who had appointed Mubariz Khan as a full-fledged viceroy of the Deccan, decided to fight Mubariz Khan. He defeated and later killed Mubariz Khan in the Battle of Shakr-Kheda (1724). He now assumed control of the Deccan. In 1725, he became the viceroy and conferred on himself the title of Asaf-Jah.

87. Consider the following statements regarding Vigyan Jyoti Scheme 1. It was launched by the Ministry of Human Resource Development. 2. It will create a level-playing field for the meritorious girls in high school to pursue Science, Technology, Engineering, and Mathematics (STEM) in their higher education. 3. The programme will cover girl students from Higher Secondary to Post Graduation. Which of the statements given above is/are correct? www.insightsonindia.com 72 Insights IAS INSTA 75 Days REVISION PLAN for Prelims 2020 - InstaTests

(a) 2 only (b) 2 and 3 only (c) 1 only (d) 1, 2 and 3

Solution: A

Vigyan Jyoti Scheme:

• Vigyan Jyoti Scheme is launched by the Department of Science & Technology (DST). • It is an initiative that will create a level-playing field for the meritorious girls in high school to pursue Science, Technology, Engineering, and Mathematics (STEM) in their higher education. • The programme will cover girl students from grade 9 to 12. • It also offers exposure for girl students from the rural background to help to plan their journey from school to a job of their choice in the field of science.

88. Consider the following statements regarding Melamine 1. It is an organic base chemical rich in nitrogen. 2. It is usually added to increase the taste of the food 3. It can cause bladder stones in animals Which of the statements given above is/are correct? (a) 1 and 3 only (b) 2 only (c) 3 only (d) 1, 2 and 3

Solution: A

• Melamine is an organic base chemical most commonly found in the form of white crystals rich in nitrogen. • It is added to water adulterated milk to increase the nitrogen content and hence its apparent protein content. • Addition of melamine into food is not approved by the FAO/WHO Codex Alimentarius (food standard commission), or by any national authorities. www.insightsonindia.com 73 Insights IAS INSTA 75 Days REVISION PLAN for Prelims 2020 - InstaTests

Health effects of melamine

• Melamine can cause bladder stones in animals. • When combined with cyanuric acid, which may also be present in melamine powder, melamine can form crystals that can give rise to kidney stones. • Melamine has also been shown to have carcinogenic effects in animals in certain circumstances.

89. Consider the following statements regarding Farrukhsiyar 1. He became the new emperor with the help of Sayyid brothers. 2. In1632, he issued the Golden Farman to English. Which of the statements given above is/are correct? (a) 1 only (b) 2 only (c) Both 1 and 2 (d) Neither 1 nor 2

Solution: A

Farrukhsiyar (1713-1719) After killing Jahandar Shah with the help of Sayyid brothers— Abdulla Khan and Hussain Ali (known as ‘King Makers’), Farrukhsiyar became the new emperor. He followed a policy of religious tolerance by abolishing Jaziya and pilgrimage tax. In 1717, he gave farmans to the British. In 1719, the Sayyid brothers, with the help of Peshwa Balaji Vishwanath, dethroned Farrukhsiyar. Later, he was blinded and killed. It was the first time in the Mughal history that an emperor was killed by his nobles. 1632: The English obtained the Golden Farman with the right to trade in the kingdom of Golkunda for a fixed customs duty, from the Sultan of Golkunda.

90. Consider the following statements regarding Battles of Panipat 1. First Battle of Panipat laid the foundation of the Mughal Empire. 2. Second Battle of Panipat laid the foundation of the Sur dynasty 3. Third Battle of Panipat was fought between the Marathas and the Mughals. Which of the statements given above is/are correct? (a) 1 only (b) 2 and 3 only

www.insightsonindia.com 74 Insights IAS INSTA 75 Days REVISION PLAN for Prelims 2020 - InstaTests

(c) 1 and 3 only (d) 1, 2 and 3

Solution: A

• The first Battle of Panipat in 1526 was between Babur and Ibrahim Lodi. The result of the battle laid the foundation of the Mughal Empire by ending the rule of the Delhi Sultanate. • The Second Battle of Panipat in 1556 was between Akbar and Hemu; it decided in favour of the continuation of the Mughal rule. • The Third Battle of Panipat in 1761 between the Marathas and Ahmad Shah Abdali put an end to the Maratha ambition of ruling over India.

91. Consider the following statements regarding National Security Act 1. It empowers the central government and state governments to detain a person to prevent him/her from acting in any manner prejudicial to the security of India. 2. The act also gives power to the governments to detain a foreigner in a view to regulate his presence or expel from the country. Which of the statements given above is/are correct? (a) 1 only (b) 2 only (c) Both 1 and 2 (d) Neither 1 nor 2

Solution: C

• The National Security Act is a stringent law that allows preventive detention for months, if authorities are satisfied that a person is a threat to national security or law and order. • The person does not need to be charged during this period of detention. The goal is to prevent the individual from committing a crime. • It empowers the Central Government and State Governments on grounds include • Acting in any manner prejudicial to the defence of India, its relations with foreign powers, its security.

www.insightsonindia.com 75 Insights IAS INSTA 75 Days REVISION PLAN for Prelims 2020 - InstaTests

• Regulating the continued presence of any foreigner in India or with a view to making arrangements for his expulsion from India. • Acting in any manner prejudicial to the security of the State or the maintenance of public order/essential supplies and services. • The act also gives power to the governments to detain a foreigner in a view to regulate his presence or expel from the country.

92. Consider the following statements regarding Danes 1. The Danish East India Company was established in 1616 and they founded a factory at Tranquebar. 2. The Danes are better known for their missionary activities than for commerce. Which of the statements given above is/are correct? (a) 1 only (b) 2 only (c) Both 1 and 2 (d) Neither 1 nor 2

Solution: C

• The Danish East India Company was established in 1616 and, in 1620, they founded a factory at Tranquebar near Tanjore, on the eastern coast of India. • Their principal settlement was at Serampore near Calcutta. • The Danish factories, which were not important at any time, were sold to the British government in 1845. • The Danes are better known for their missionary activities than for commerce.

93. Consider the following statements regarding Joseph Francis Dupleix 1. Dupleix was the first European to interfere in the internal politics of the Indian rulers. 2. Dupleix was the originator of the practice of subsidiary alliance in India. Which of the statements given above is/are correct? (a) 1 only (b) 2 only (c) Both 1 and 2

www.insightsonindia.com 76 Insights IAS INSTA 75 Days REVISION PLAN for Prelims 2020 - InstaTests

(d) Neither 1 nor 2

Solution: C

• Joseph Francis Dupleix, born in 1697, was the son of a wealthy Farmer-General of Taxes and Director-General of the Company of the Indies. He got a high post at Pondicherry in 1720, allegedly on the basis of influence of his father. At Pondicherry he made a lot of money by private trade, which was then permitted to servants of the French company. In 1741, he was appointed as the Director-General of French colonies in India. Later, he was conferred the title of Nawab by the Mughal emperor and the Subahdar of Deccan, Muzzaffar Jang. • Dupleix was the first European to interfere in the internal politics of the Indian rulers. • Dupleix was, in fact, the originator of the practice of subsidiary alliance in India. He placed a French army at Hyderabad at the expense of the subahdar.

94. Consider the following statements 1. Defence Acquisition Council is chaired by Prime Minister. 2. Nirbhay is India’s first indigenously produced cruise missile. 3. Bold Kurukshetra is a joint military exercise between India and France. Which of the statements given above is/are correct? (a) 2 only (b) 2 and 3 only (c) 1 only (d) None

Solution: A

The Defence Acquisition Council is the highest decision-making body in the Defence Ministry for deciding on new policies and capital acquisitions for the three services (Army, Navy and Air Force) and the Indian Coast Guard.

• The Minister of Defence is the Chairman of the Council. • It was formed, after the Group of Ministers recommendations on ‘Reforming the National Security System’, in 2001, post Kargil War (1999).

www.insightsonindia.com 77 Insights IAS INSTA 75 Days REVISION PLAN for Prelims 2020 - InstaTests

Nirbhay is a long range, all-weather, subsonic cruise missile designed and developed in India by the Aeronautical Development Establishment (ADE) which is under Defence Research and Development Organisation (DRDO).

• Nirbhay is India’s first indigenously produced cruise missile. • The missile can be launched from multiple platforms and is capable of carrying conventional and nuclear warheads. It is currently under development and undergoing flight trials. The 12th edition of joint military exercise between India and Singapore, BOLD KURUKSHETRA 2019, culminated on 11 April 2019 after an impressive closing ceremony held at Babina Military Station. The four day long joint training focused on developing interoperability and conduct of joint tactical operations in mechanised warfare. The troops learnt about each other’s organisations and best practices being followed in combat. https://pib.gov.in/Pressreleaseshare.aspx?PRID=1570421

95. Consider the following statements regarding English merchants 1. Queen Elizabeth I issued a charter with rights of exclusive trading to a company in 1600. 2. Initially, a monopoly was granted for indefinite period, which was later restricted to fifteen years by a fresh charter. 3. Captain Hawkins arrived in the court of Aurangzeb for requesting trade rights. Which of the statements given above is/are Not correct? (a) 1 only (b) 2 and 3 only (c) 1 and 3 only (d) 1, 2 and 3

Solution: B

In 1599, a group of English merchants calling themselves the ‘Merchant Adventurers’ formed a company. On December 31, 1600, Queen Elizabeth I issued a charter with rights of exclusive trading to the company named the ‘Governor and Company of Merchants of London trading into the East Indies’. Initially, a monopoly of fifteen years was granted, which in May 1609 was extended indefinitely by a fresh charter. Captain Hawkins arrived in the court of Jahangir in April 1609 itself. But the mission to establish a factory at Surat didn’t succeed due to opposition from the Portuguese, and Hawkins left Agra in November 1611.

www.insightsonindia.com 78 Insights IAS INSTA 75 Days REVISION PLAN for Prelims 2020 - InstaTests

96. Consider the following statements regarding Mosquirix 1. It is the world’s first licensed malaria vaccine 2. It is can be used against parasitic disease of any kind. 3. India is instrumental in developing this vaccine. Which of the statements given above is/are correct? (a) 3 only (b) 1 and 2 only (c) 1 only (d) 2 and 3 only

Solution: B

RTS,S/AS01 (trade name Mosquirix) is a recombinant protein-based malaria vaccine.

• Approved for use by European regulators in July 2015, it is the world’s first licensed malaria vaccine and also the first vaccine licensed for use against a human parasitic disease of any kind. • The RTS,S vaccine was conceived of and created in the late 1980s by scientists working at SmithKline Beecham Biologicals (now GlaxoSmithKline Vaccines) laboratories in Belgium. • The vaccine was further developed through a collaboration between GSK and the Walter Reed Army Institute of Research and has been funded in part by the PATH Malaria Vaccine Initiative and the Bill and Melinda Gates Foundation. Its efficacy ranges from 26 to 50% in infants and young children. • India didn’t play any significant role in developing the vaccine.

97. Consider the following statements regarding Alfonso de Albuquerque 1. Albuquerque was the first Portuguese governor in India. 2. He was considered as the real founder of the Portuguese power in the East. 3. One of the notable features under his rule was the abolition of sati practice. Which of the statements given above is/are correct? (a) 1 and 2 only (b) 2 and 3 only (c) 1 and 3 only (d) 1, 2 and 3

www.insightsonindia.com 79 Insights IAS INSTA 75 Days REVISION PLAN for Prelims 2020 - InstaTests

Solution: B

Alfonso de Albuquerque

• Albuquerque, who succeeded Almeida as the Portuguese governor in India, was the real founder of the Portuguese power in the East, a task he completed before his death. He secured for Portugal the strategic control of the Indian Ocean by establishing bases overlooking all the entrances to the sea. There were Portuguese strongholds in East Africa, off the Red Sea, at Ormuz; in Malabar; and at Malacca. The Portuguese, under Albuquerque bolstered their stranglehold by introducing a permit system for other ships and exercising control over the major ship-building centres in the region. The nonavailability of timber in the Gulf and Red Sea regions for ship- building also helped the Portuguese in their objectives. • Albuquerque acquired Goa from the Sultan of Bijapur in 1510 with ease; the principal port of the Sultan of Bijapur became “the first bit of Indian territory to be under the Europeans since the time of Alexander the Great”. An interesting feature of his rule was the abolition of sati.

98. Consider the following statements regarding The World Food Programme (WFP) 1. It is the food-assistance branch of the International Fund for Agricultural Development. 2. WFP works to help people who cannot produce or obtain enough food for themselves and their families. Which of the statements given above is/are correct? (a) 1 only (b) 2 only (c) Both 1 and 2 (d) Neither 1 nor 2

Solution: B

The World Food Programme (WFP) is the food-assistance branch of the United Nations and the world’s largest humanitarian organization addressing hunger and promoting food security. According to the WFP, it provides food assistance to an average of 91.4 million people in 83 countries each year.

• From its headquarters in Rome and from more than 80 country offices around the world, the WFP works to help people who cannot produce or obtain enough food for

www.insightsonindia.com 80 Insights IAS INSTA 75 Days REVISION PLAN for Prelims 2020 - InstaTests

themselves and their families. It is a member of the United Nations Development Group and part of its executive committee • WFP was established in 1961 after the 1960 Food and Agriculture Organization (FAO) Conference, when George McGovern, director of the US Food for Peace Programmes, proposed establishing a multilateral food aid programme. The WFP was formally established in 1963 by the FAO and the United Nations General Assembly on a three- year experimental basis. In 1965, the programme was extended to a continuing basis.

99. Consider the following statements regarding Dutch invasion 1. The Dutch founded their first factory in Nagapatam in 1605. 2. The battle of Hooghly dealt a crushing blow to Dutch ambitions in India. Which of the statements given above is/are correct? (a) 1 only (b) 2 only (c) Both 1 and 2 (d) Neither 1 nor 2

Solution: B

• Commercial enterprise led the Dutch to undertake voyages to the East. Cornelis de Houtman was the first Dutchman to reach Sumatra and Bantam in 1596. In 1602, the States-General of the Netherlands amalgamated many trading companies into the East India Company of the Netherlands. • This company was also empowered to carry on war, to conclude treaties, to take possession of territory and to erect fortresses. Dutch Settlements

• After their arrival in India, the Dutch founded their first factory in Masulipatnam (in Andhra) in 1605. They went on to establish trading centres at different parts of India and thus became a threat to the Portuguese. They captured Nagapatam near Madras (Chennai) from the Portuguese and made it their main stronghold in South India. • The Dutch got drawn into the trade of the Malay Archipelago. Further, in the third Anglo-Dutch War (1672-74), communications between Surat and the new English settlement of Bombay got cut due to which three homebound English ships were captured in the Bay of Bengal by the Dutch forces. The retaliation by the English resulted in the defeat of the Dutch, in the battle of Hooghly (November 1759), which dealt a crushing blow to Dutch ambitions in India.

www.insightsonindia.com 81 Insights IAS INSTA 75 Days REVISION PLAN for Prelims 2020 - InstaTests

100. Consider the following statements regarding ExoMars 1. It is a joint endeavour between ESA and Roscosmos. 2. It aims to address the question of whether life has ever existed on Mars. Which of the statements given above is/are correct? (a) 1 only (b) 2 only (c) Both 1 and 2 (d) Neither 1 nor 2

Solution: C

• Launch of ExoMars rover delayed to 2022. About the rover:

• The European-built Rosalind Franklin rover, named for the famed British chemist and X-ray crystallographer whose work contributed to DNA research, recently passed final pre-launch thermal and vacuum tests at an Airbus facility in Toulouse, France. • Rosalind Franklin is the first European Mars rover. About ExoMars:

• It is a joint endeavour between ESA and the Russian space agency, Roscosmos. • The primary goal of the ExoMars programme is to address the question of whether life has ever existed on Mars. Components of the mission: The programme comprises two missions.

• The first launched in March 2016 and consists of the Trace Gas Orbiter (TGO) and Schiaparelli, an entry, descent and landing demonstrator module. • TGO’s main objectives are to search for evidence of methane and other trace atmospheric gases that could be signatures of active biological or geological processes. The Schiaparelli probe crashed during its attempt to land on Mars. • The second, comprising a rover and surface platform, is planned for 2022. Together they will address the question of whether life has ever existed on Mars.

www.insightsonindia.com 82 Insights IAS INSTA STORIE S

😊

www.insightsonindia.com INSIGHTS IAS 

www.insightsonindia.com INSIGHTS IAS